SlideShare a Scribd company logo
1 of 21
1. A human body cools in water much                 C. Mature
faster that in the air. What way of heat emission   D. Atretic
in water is much more efficient?                    E. Primary
A. Convection                                       7. 2 hours after a skeletal extension was
B. Heat radiation                                   performed to a 27 year old patient with
C. Heat conduction                                  multiple traumas (closed injury of chest
D. Sweat evaporation                                closed fracture of right thigh) his condition
E. -                                                abruptly became worse and the patient died
2. A driver who got a trauma in a road accident     from acute cardiopulmonary decompensation.
and is shocked has reduction of daily               Histological examination of pulmonary
urinary output down to 300 m1. What is the          and cerebral vessels stained with Sudan III
main pathogenetic factor of such diuresis           revealed orange drops occluding the vessel
change?                                             lumen. What complication of polytrauma
A. Drop of arterial pressure.                       was developed?
B. Secondary hyperaldosteronism                     A. Air embolism
C. Increased vascular permeability                  B. Thromboembolism
D. Decreased number of functioning                  C. Fat embolism
glomerules                                          D. Microbal embolism
E. Drop of oncotic blood pressure                   E. Gaseous embolism
3. Myocyte cytoplasm contains a big number          8. A patient complains of frequent diarrheas,
of dissolved metabolites of glucose oxidation.      especially after consumption of
Name one of them that turns directly                rich food, weight loss. Laboratory examination
into a lactate:                                     revealed steatorrhea; his feces were
A. Pyruvate                                         hypocholic. What might have caused such
B. Glycerophosphate                                 condition?
C. Glucose 6-phosphate                              A. lnflammation of mucous membrane of
D. Oxaloacetate                                     small intestine
E. Fructose 6-phosphate                             B. Obturation of biliary tracts
4. Continuous taking of some drugs foregoing        C. Lack of pancreatic lipase
the pregnancy increase the risk of giving           D. Lack of pancreatic phospholipase
birth to a child with genetic defects. What is      E. Unbalanced diet
this effect called?                                 9. A 12 year old child has intolerance to
A. Mutagenic effect                                 some foodstuffs. Their consumption causes
B. Blastomogenic effect                             an allergic reaction in form of itching skin
C. Fetotoxical effect                               eruptions. What antihistaminic drug should
D. Embryotoxic effect                               be admistered so that the child could attend
E. Teratogenic effect                               school?
5. A patient has yellow skin colour, dark urine     A. Loratadine
dark-yellow feces. What substance will              B. Dimedrol
have strengthened concentration in the              C. Ephedrine
blood scrum?                                        D. Aminophylline
A. Verdoglobin                                      E. Diclofenac
B. Conjugated bilirubin                             10. Labelled amino acids alanine and
C. Biliverdin                                       tryptophane were injected to a mouse
D. Unconjugated bilirubin                           in order to study localization of protein
E. Mesobilirubin                                    synthesis in its cells. The labelled amino acids
6. Examination of an ovary specimen stained         will be accumulated near the following
by hematoxylin-eosine revealed a follicle           organellas:
in which follicular epithelium consisted            A. Smooth endoplasmic reticulum
of 1-2 layers of cubic cells. There was also        B. Ribosomes
a bright red membrane around the ovocyte.           C. Lysosomes
What follicle is it?                                D. Cell centre
A. Secondary                                        E. Golgi apparatus
B. Primordilal                                      11. As a result of prophylactic meedical
examination a 7 year old boy was diagnosed        B. Intensification of glucose absorption m
with Lesch-Nyhan syndrome (only boys fall         the bowels
ill with it). The boy's parents are healthy but   C. Gluconeogenesis activation
his grandfather bv his mother's side suffers      D. Glycogenogenesis activation
from the same disease. What type or disease       E. Inhibition of glycogen synthesis
inheritance is it?                                16. A man with a stab wound in the area of
A. Dominant, sex-linked                           quadrilateral foramen applied to a doctor.
B. Recessive, sex-linked                          Examination revealed that the patient was
C. Autosomal recessive                            unable to draw his arm aside from his body.
D. Autosomal dominant                             What nerve is most probably damaged?
E. Semidominance                                  A. N.axillaris
12. A patient ill with chronic cardiac            B. N.radialis
insufficiency was prescribed an                   C. N.subclavius
average therapeutic dose of digoxin. Two          D. Nmedianus
weeks after begin of its taking there             E. N ulnaris
appeared symptoms of drug intoxication            17. Ultramicroscopical examination of
(bradycardia, extrasystole, nausea). Name         "dark" hepatocyte population in the cell
the phenomenon that caused accumulation           cytoplasm detected a developed granular
of the drug in the organism?                      endoplasmic reticulum. What function has
A. Material cumulation                            this organella in these cells?
B. Tolerance                                      A. Bile production
C. Functional cumulation                          B. Deintoxicative function
D. Idiosyncrasy                                   C. Calcium ion depositing
E. Tachyphylaxis                                  D. Carbohydrate synthesis
13. Bacterioscopic examination of a smear         E. Synthesis of blood plasma proteins
from the pharynx of a diphtheria suspect          18. A 17 year old boy fell seriously ill, the
revealed bacilli with volutine granules. What     body temperature rose up to 38,5°C, there
etiotropic drug should be chosen in this          appeared cough, rhinitis, lacrimation, nasal
case?                                             discharges. What inflammation is it?
A. Eubiotic                                       A. Hemorrhagic
B. Antidiphtheritic antitoxic serum               B. Purulent
C. Bacteriophage                                  C. Fibrinous
D. Diphtheritic anatoxin                          D. Catarrhal
E. Interferon                                     E. Serous
14. A culture of monkey cells (Vero) and          19. Researchers isolated 5 isoenzymic forms
a group of mouse sucklings were infected          of lactate dehydrogenase from the human
with an inoculum taken from a child with          blood serum and studied their properties.
provisional diagnosis "enterovirus infection".    What property indicates that the isoenzymic
There was no cytopathic effect on the             forms were isolated from the same enzyme?
cell culture but mouse sucklings died. What       A. The same electrophoretic mobility
enteric viruses might have caused disease of      B. Tissue localization
this child?                                       C. The same molecular weight
A. Coxsackie B                                    D. The same physicochemical properties
B. Coxsackie A                                    E. Catalyzation of the same reaction
C. Unclassified enteric viruses 68-71             20. A 60 year old patient has impaired
D. ECHO virus                                     perception of high-frequency sounds. These
E. Polioviruses                                   changes were caused by damage of the
15. A patient ill with neurodermatitis has        following auditory analyzer structures:
been taking prednisolone for a long time.         A. Tympanic membrane
Examination revealed high rate of sugar in        B. Main cochlea membrane near the oval
his blood. This complication is caused by         window
the drug influence upon the following link        C. Eustachian tube
of carbohydrate metabolism:                       D. Middle ear muscles
A. Activation of insulin decomposition            E. Main cochlea membrane near the helicotrema
21. ECG of a patient with hyperfunction of          D. Biotin
thyroid gland showed heart hurry. It is indicated   E. Vitamin A
by depression of the following ECG                  26. A patient ill with essential hypertension
element:                                            was recommended a drug that prevents
A. QRS complex                                      thrombosis. It is to be taken parenterally.
B. R - R interval                                   What drug is it?
C. P – Q interval                                   A. Protamine sulfate
D. P – Q segment                                    B. Amben
E. P - T interval                                   C. Heparin
22 A patient taking clonidine for essential         D. Neodicumarin
hypertension treatment was using alcohol            E. Syncumar
that caused intense inhibition of central           27. A 38 year old patient suffers from
nervous system. What may it be connected            rheumatism in its active phase. What
with?                                               laboratory characteristic of blood serum
A. Intoxication                                     is of diagnostic importance in case of this
B. Idiosyncrasy                                     pathology?
C. Cumulation                                       A. Uric acid
D. Effect potentiating                              B. Creatinine
E. Effect summation                                 C. C-reactive protein
23. A patient is ill with diabetes mellitus         D. Transferrin
accompanied by hyperglycemia on                     E. Urea
an empty stomach (7,2 millimole/l). The             28. A 28 year old man had a gunshot wound
hyperglycemia rate can be retrospectively           of shin that resulted in an ulcer from the side
estimated (over the last 4-8 weeks before           of the injury. What is the main factor of
the examination) on the ground of the rate          neurodystrophy pathogenesis in this case?
of the following blood plasma protein:              A. Microcirculation disturbance
A.Albumin                                           B. Infection
B. Ceruloplasmin                                    c. Tissue damage
C. Fibrinogen                                       D. Psychical stress
D. Glycated hemoglobin                              E. Traumatization of peripheral nerve
E. C-reactive protein                               29. A 26 year old man is in the torpid shock
24. 6 months after labour a woman had               Phase as a result of a car accident. In blood:
uterine hemorrhage. Gynaecological examination      3,2· 109/1. What is the leading mechanism of
of uterine cavity revealed a dark-red               leukopenia development?
tissue with multiple cavities resembling of         A. Leikopoiesis inhibition
a "sponge". Microscopic examination of a            B. Redistribution of leukocytes m
tumour revealed in blood lacunas atypic light       bloodstream
epithelial Langhans cells and giant cells           C. Disturbed going out of mature leukocytes
of syncytiotrophoblast. What tumour is it?          from the marrow into the blood
A. Adenocarcinoma                                   D. Lysis of leukocytes in the blood-forming
B. Cystic mole                                      organs
C. Fibromyoma                                       E. Intensified elimination of leukocytes from
D. Chorioepithelioma                                the organism
E. Squamous cell nonkeratinous carcinoma            30. Parents of a 3 year old child have
25. A 3 year old child with symptoms of             been giving him antibiotics with purpose of
stomatitis, gingivitis and dermatitis of open       preventing enteric infections for a long time.
skin areas was delivered to a hospital.             A month later the child's condition changed
Examination revealed inherited disturbance          for the worse. Blood examination revealed
of neutral amino acid transporting in the           apparent leukopenia and granulocytopenia.
bowels. These symptoms were caused by the           What is the most probable mechanism of
deficiency of the following vitamin:                blood changes?
A. Niacin                                           A. Autoimmune
B. Pantothenic acid                                 B. Myelotoxic
C. Cobalamin                                        C. Redistributive
D. Age-specific                                      traumatology department. Objectively: cut
E. Hemolytic                                         wound of palmar surface of left hand: middle
31. Examination of a 12 year old boy with            phalanxes of II-V fingers don't bend.
developmental lag revealed achondroplasia:           What muscles are damaged?
shortening of upper and lower limbs as a             A. Palmar interosseous muscles
result of growth disorder of epiphyseal cartilages   B. Profound finger flexor
of long tubal bones. This disease is:                C. Dorsal interosseous muscles
A. Acquired                                          D. Lumbrical muscles
B. Inherited, dominant                               E. Superficial finger flexor
C. Inherited, sex-linked                             36. A 65 year old man suffering from
D. Congenital                                        gout complains of kidney pain. Ultrasound
E. Inherited, recessive                              examination revealed renal calculi. The
32. Hartnup disease is caused by point               most probable cause of calculi formation
mutation of only one gene which results in           is the strengthened concentration of the
disturbance of tryptophane absorption in             following substance:
the bowels and its resorption in the renal           A. Uric acid
tubules. It is the reason for disorder of            B. Cholesterol
both digestive and urination systems. What           C. Bilirubin
genetic phenomenon is observed in this               D. Urea
case?                                                E. Cystine
A. Polymery                                          37. A patient with frequent attacks of
B. Complementary interaction                         stenocardia was prescribed sustak-forte to
C. Pleiotropy                                        be taken one tablet twice a day. At first
D. Semidominance                                     the effect was positive but on the second
E. Codominance                                       day stenocardia attacks resumed. What can
33. A 46 year old patient applied to a doctor        explain inefficiency of the prescribed drug?
complaining about joint pain that becomes            A. Tachyphylaxis
stronger the day before weather changes.             B. Sensibilization
Blood examination revealed strengthened              C. Dependence
concentration of uric acid. The most                 D. Cumulation
probable cause of the disease is the intensified     E. Idiosyncrasy
disintegration of the following substance:           38. An aged man had raise of arterial
A. Uridine triphosphate                              pressure under a stress. It was caused by
B. Thymidine monophosphate                           activation of:
C. Uridine monophosphate                             A. Functions of thyroid gland
D. Adenosine monophosphate                           B. Sympathoadrenal system
E. Cytidine monophosphate                            C. Functions of adrenal cortex
34. A 28 year old patient had high arterial          D. Hypophysis function
pressure, hematuria and facial edemata. In           E. Parasympathetic nucleus of vagus
spite of treatment renal insufficiency was           39. A cerebral trauma caused increased
progressing. 6 months later the patient died         ammonia generation. What amino acid
from uremia. Microscopic examination                 participates in the excretion of ammonia
of his kidneys and their glomerules revealed         from the cerebral tissue?
proliferation of capsule nephrothelium and           A. Glutamic
of podocytes with "demilune "formation,              B. Lysine
sclerosis and hyalinosis of glomerules. What         C. Tyrosine
disease corresponds with the described picture?      D. Tryptophan
A. Acute glomerulonephritis                          E. Valine
B. Subacute glomerulonephritis                       40. As a result of posttranslative modifications
C. Nephrotic syndrome                                some proteins taking part in blood
D. Chronic glomerulonephritis                        coagulation, particularly prothrombin,
E. Acute pyelonephritis                              become capable of calcium binding. The
35. A 35 year old man with a trauma                  following vitamin takes part in this process:
of his left hand was admitted to the                 A. B2
B. B1                                                  46. While palpating mammary gland of a
C. A                                                   patient a doctor revealed an induration
D. C                                                   in form of a node in the inferior medial
E. K                                                   quadrant. Metastases may extend to the
41. A student takes notes of a lecture. Quality        following lymph nodes:
of his notes became significantly worse when his       A. Superior diaphragmal
neighbours began talking. What                         B. Profound lateral cervical
type of conditional reflex inhibition was the          C. Bronchopulmonary
cause of it?                                           D. Posterior mediastinal
A. Protective                                          E. Parasternal
B. Differentiated                                      47. A gynaecologist was examining a patient
C. External                                            and revealed symptoms of genital tract
D. Delayed                                             inflammation. A smear from vagina contains
E. Extinctive                                          pyriform protozoa with a spine, flagella
42. A 7 vear old child is ill with bronchilis. It is   at their front: there is also an undulating
necessary to administer him an                         membrane. What disease can be suspected?
antibacterial drug. What drug of fluoroquinolone       A. Intestinal trichomoniasis
group is CONTRA-INDICATED at                           B. Toxoplasmosis
this age?                                              C. Lambliasis
A. Cyprofloxacin                                       D. Urogenital trichomoniasis
B. Sulfadimethoxine                                    E. Balantidiasis
C. Ampiox                                              48. In course of an experiment a big number
D. Ampicillin                                          of stem cells of red bone marrow was in
E. Amoxicillin                                         some way destructed. Regeneration of which
43. A patient is ill with hepatocirrhosis.             cell populations in the loose connective
State of antitoxic liver function can be               tissue will be inhibited?
characterized by examination of the following          A. Of lipocytes
substance exreted by urine:                            B. Of fibroblasts
A. Uric acid                                           C. Of pericytes
B. Hippuric acid                                       D. Of pigment cells
C. Ammonium salts                                      E. Of macrophags
D. Amino acids                                         49. A pregnant woman had her blood group
E. Creatinine                                          identified. Reaction of erythrocyte agglutination
44. A patient suffering from chronic cardiac           with standard serums of 0αβ (I), Bα
insufficiency was recommended to undergo               (III) groups didn't proceed with standard
a prophylactic course of treatment with a              serum of Aβ (II) group. The blood group
cardiological drug from the group of cardiac           under examination is:
glycosides that is to be taken enterally. What         A. Aβ (II)
drug was recommended')                                 B. AB (IV)
A. Corglycon                                           C. Bα (III)
B. Cordiamin                                           D. -
C. Strophanthine                                       E. 0αβ(I)
D. Cordarone                                           50. Prophylactic medical examination of a
E. Digoxin                                             36 year old driver revealed that his AP was
45. 48 hours after tuberculine test (Mantoux           150/90 mm Hg. At the end of working day he
test) a child had a papule 10 mm in diameter           usually hears ear noise, feels slight indisposition
on the spot of tuberculine injection. What             that passes after some rest. He was diagnosed
hypersensitivity mechanism underlies these             with essential hypertension. What
changes?                                               is the leading pathogenetic mechanism in
A. Anaphylaxy                                          this case?
B. Granulomatosis                                      A. Neurogenetic
C. Antibody-dependent cytotoxicity                     B. Endocrinal
D. Cellular cytotoxicity                               C. Reflexogenic
E. Immunocomplex cytotoxicity                          D. Nephric
E. Humoral                                        "progressing muscular dystrophy" got his
51. A histological specimen of spleen             urine tested. What compound will confirm
shows a vessel with a wall consisting             this diagnosis if found in urine?
of endothelium and subendothelial layer,          A. Kreatine
median membrane is absent, exterior               B. Calmodulin
membrane inosculates with the layers of           C. Collagen
spleen connective tissue. What vessel is it?      D. Myoglobin
A. Artery of muscular type                        E. Porphyrin
B. Capillary                                      57. While playing volleyball a sportsman
C. Vein of muscular type                          made a jump and landed on the outside
D. Arteriole                                      edge of his foot. He felt acute pain in the
E. Vein of non-muscular type                      talocrural joint, active movements are limited,
52. It was revealed that T-lymphocytes            passive movements are unlimited but
were affected by HIY. Virus enzyme reverse        painful. A bit later there appeared a swelling
transcriptase (RN A -dependent                    in the area of external ankle, the skin
DNA polymerase) - catalyzes the synthesis         became red and warm. What type of peripheral
of:                                               circulation disturbance is the case?
A. Viral DNA on DNA matrix                        A. Arterial hyperemia
B. Virus informational RNA on the matrix of       B. Venous hyperemia
DNA                                               C. Stasis
C. DNA on virus ribosomal RNA                     D. Embolism
D. DNA on the matrix of virus mRNA                E. Thrombosis
E. mRNA on the matrix of virus protein            58. Autopsy of a man who had been working
53. Planned mass vaccination of all newborn       as a miner for many years and died from
5-7 day old children against tuberculosis plays   cardiopulmonary decompensation revealed
an important role in tuberculosis prevention.     that his lungs were airless, sclerosed, their
In this case the following vaccine is             apexex had emphysematous changes, the
applied:                                          lung surface was greyish-black, the incised
A.BCG                                             lung tissue was coal-black. What disease
B. Adsorbed diphtheria vaccine                    caused death?
C. Diphteria and tetanus toxoids and              A. Silicosis
pertussis vaccine                                 B. Talcosis
D. Diphtheria and tetanus anatoxin vaccine        C. Anthracosis
E. -                                              D. Aluminosis
54. Life cycle of a cell includes the process     E. Asbestosis
of DNA autoreduplieation. As a result of it       59. Examination of a 55 year old woman
monochromatid chromosomes turn into               revealed under the skin of submandibular
bichromatid                                       area a movable slowly growing pasty
ones. What period of cell cycle                   formation with distinct borders l,0x0,7 cm
does this phenomenon fall into?                   large. Histological examination revealed
A. G2                                             Iipocytes
B. Go                                             that form segments of diffrent forms
C. G1                                             and sizes separated from each other by
D. S                                              thin layers of connective tissue with vessels.
E. M                                              What is the most probable diagnosis?
55. Rest potential of a cell equals -80 mV.       A. Lipoma
At what stage of action potential did the         B. Fibroma
membrane potential equal +30 mV?                  C. Liposarcoma
A. After depolarization                           D. Fibrosarcoma
B. After hyperpolarization                        E. Angioma
C. Reverse polarization                           60. A patient is 44 years old. Laboratory
D. -                                              examination of hrs blood revealed that
E. Depolarization                                 content of proteins in plasma was 40 g/l.
56. A patient with suspected diagnosis            What influence will be exerted on the
transcapillary water exchange?                       C. M.rhomboideus minor
A. Exchange will stay unchanged .                    D. M.latissimus dorsi
B. Filtration will be increased, reabsorption-       E. M.sternocleidomastoideus
decreased                                            65. A patient with a knife wound in the
C. Filtration will be decreased, reabsorption        left lumbal part was delivered to the
- increased                                          emergency hospital. In course of operation
D. Both filtration and reabsorption will be          a surgeon found that internal organs were
increased                                            not damaged but the knife injured one of
E. Both filtration and reabsorption will be          muscles of renal pelvis. What muscle is it?
decreased                                            A. Iliac muscle
61. A 30 year old woman has face edemata.            B. Abdominal internal oblique muscle
Examination revealed proteinuria (5,87 g/l),         C. Erector muscle of spine
hypoproteinemia, dysproteinemia,                     D. Greater psoas muscle
hyperlipidemia.                                      E. Abdominal external oblique muscle
What condition is the set of these                   66. A 56 year old patient came to a hospital
symptoms typical for?                                with complaints about general weakness,
A. Chronic renal failure                             tongue pain and burning, sensation of limb
B. Nephritic syndrome                                numbness. In the past he underwent resection of
C. Nephrotic syndrome                                forestomach. In blood: Hb- 80 g/l;
D. Chronic pyelonephritis                            erythrocytes – 2,0*1012/l; colour index - 1.2,
E. Acute renal failure                               leukocytes – 3,5*109/l. What anemia type is
62. Vitamin B1 deficiency results in disturbance     it?
of oxidative decarboxylation of α-ketoglutaric       A. Hemolytic
acid. This will disturb synthesis                    B. B12-folate deficient
of the following cocnzyme:                           C. Aplastic
A. Nicotinamide adenine dinucleotide                 D. Posthemorrhagic
(NAD)                                                E. Iron-deficient
B. Lipoic acid                                       67. A patient ill with thrombophlehitis of his
C. Flavine adenine dinucleotide (FAD)                lower limbs had chest pain, blood spitting,
D. Thiamine pyrophosphate                            progressing respiratory insufficiency that led
E. Coenzyme A                                        to his death. Autopsy diagnosed multiple
63. Skin of a man who died from cardiac              lung a infarctions. What is the most probable
insufficiency has an eruption in form                cause of their development?
of spots and specks. There are also                  A. Thromboembolism of pulmonary artery
bedsores in the area of sacrum and spinous           branches
vertebral processes. Microscopical                   B. Thrombosis of bronchial arteries
examination of CNS, skin, adrenal glands             C. Thrombosis of pulmonary artery branches
revealed in the vessels of microcirculatory          D. Thrombosis of pulmonary veins
bed and in small arteries destructiveproliferative   E. Thromboembolism of bronchial arteries
endothrombovasculitis with                           68. A concentrated solution of sodium
Popov's granulomas; interstitial myocarditis.        chloride was intravenously injected to an
What diagnosis corresponds with the described        animal. This caused decreased reabsorption
picture?                                             of sodium ions in the renal tubules. It is the
A. Spotted fever                                     result of the following changes of hormonal
B. Q fever                                           secretion:
C. Nodular periarteritis                             A. Aldosterone increase
D. Enteric fever                                     B. Reduction of atrial natriuretic factor
E. HIV                                               C. Vasopressin increase
64. A man with an injury of the dorsal area          D. Aldosterone reduction
of his neck was admitted to the resuscitation        E. Vasopressin reduction
department. What muscle occupies this                69. A patient had hemorrhagic stroke. Blood
area?                                                examination revealed strengthened kinin
A. M.trapezius                                       concentration.The patient was prescribed
B. M.scalenus anterior                               contrical. It was administered in order to
inhibit the following proteinase:                     heart structure:
A. Trypsin                                            A. Ventricle
B. Chemotrypsin                                       B. Sinoatrial node
C. Pepsin                                             C. Atrioventricular node
D. Collagenase                                        D. His' bundle
E. Kallikrein                                         E. Atrium
70. Packed cell volume of a man was 40%               76. Examination of coronary arteries
before the trauma. What packed cell volume            revealed atherosclerotic calcified plaques
will be observed 24 hours after blood loss of         closing vessel lumen by 1/3. The muscle has
750 ml?                                               multiple whitish layers of connective tissue.
A.40%                                                 What process was revealed in the myocardium?
B.30%                                                 A. Postinfarction cardiosclerosis
C.55%                                                 B. Myocardium infarction
D.50%                                                 C. Diffusive cardiosclerosis
E.45%                                                 D. Myocarditis
71. On the 2-3rd day after stomach resection          E. Tiger heart
intestinal peristalsis wasn't restored. What          77. A patient with cholelithiasis fell ill with
is to be administered for stimulation of              mechanic jaundice. Examination revealed
gastrointestinal tract?                               that the stone was in the common bile
A. Atropine sulfate                                   duct. What bile-excreting ducts make up the
B. Prasosin                                           obturated duct?
C. Acetylcholine                                      A. Ductus hepaticus sinister et ductus cysticus
D. Proserin                                           B. Ductus hepaticus dexter et sinister
E. Cyclodole                                          C. Ductus hepaticus communis et ductus
72. An ophthalmologist used a 1% mesaton              choledochus
solution for the diagnostic purpose (pupil dilation   D. Ductus hepnriclls C0l11111W1lS et ductus
for eve-ground examination). What is                  cllOledochlls
the cause of mydriasis induced by the drug?           E. Ductus hepaticus dexter ct ductus cysticus
A. Activation of α1 adrenoreceptors                   78. 2 years ago a patient underwent resection
B. Activation of β1 adrenoreceptors                   of pyloric part of stomach. He complains
C. Activation of M-cholinoreceptors                   of weakness, periodical dark shadows
D. Block of α1 adrenoreceptors                        beneath his eyes, dyspnea. In blood: Hb - 70
E. Activation of α2 adrenoreceptors                   g/l erythrocytes - 3,0 * 1012 /1, colour index
73. A patient with disturbed cerebral circulation     - 0,7. What changes of erythrocytes in blood
has problems with deglutition.                        smears are the most typicat for this condition?
What part of brain was damaged?                       A. Macrocytes
A. Interbrain                                         B. Ovalocytes
B. Cervical part of spinal cord                       C. Megalocytes
C. Midbrain                                           D. Schizocytes
D. Brainstem                                          E. Microcytes
E. Forebrain                                          79. A 6 year old child was delivered to a
74. A 16 year old boy after an illness has            hospital. Examination revealed that the child
diminished function of protein synthesis in           couldn't fix his eves, didn't keep his eyes
liver as a result of vitamin K deficiency. It         on toys, eye ground had the cherry-red spot
will cause disturbance of:                            sign. Laboratory analyses showed that brain,
A. Blood coagulation                                  Iiver and spleen had high rate of ganglioside
B. Anticoagulant generation                           glycometide. What congenital disease is the
C. Erythropoietin secretion                           child ill with?
D. Erythrocyte sedimentation rate                     A. Wilson's syndrome
E. Osmotic blood pressure                             B. MacArdle disease
75. An isolated cell of human heart                   C. Niemann-Pick disease
automatically generates excitement                    D. Tay-Sachs disease
impulses with frequency of 60 times per minute.       E. Turner's syndrome
This cell was taken from the following                80. In course of an experiment a peripheral
section of vagus of an expiremental animal          B. Retinol acetate
is being stimulated. What changes will be           C. Levamisole
observed?                                           D. Prednisolone
A. Heart rate fall                                  E. Retabolil
B. Heart hurry                                      86. A stomatologist injected a patient with
C. Increase of respiration rate                     a certain drug in order to reduce salivation
D. Bronchi dilation                                 during tooth filling. What drug is it?
E. Pupil dilation                                   A. Mesaton
81. A patient with bacterial pneumonia              B. Proserin
was prescribed benzylpenicillin. What is the        C. Atropine sulfate
mechanism of its antibacterial effect?              D. Adrenaline hydrochloride
A. Abnormal permeability of cytoplasmic             E. Pilocarpine hydrochloride
membrane                                            87. Vagus branches that innervate heart are
B. Inhibition of SH-groups of microorganism         being stimulated in course of an experiment.
enzymes                                             As a result of it the excitement conduction
C. Inhibition of synthesis of microorganism         from atria to the ventricles was brought to a
wall                                                stop. It is caused by electrophysical changes
D. Inhibition of intracellular protein              in the following structures:
synthesis                                           A. His' bundle
E. Antagonism with p-amino-benzoic acid             B. Atria
82.Blood minute volume of a 30 year old woman       C. Atrioventricular node
at rest is 5 l/m. What blood volume                 D. Sinoatrial node
is pumped through the pulmonary vessels             E. Ventricles
per minute?                                         88. A doctor examined a child and revealed
A. 3,75 l                                           symptoms of rachitis. Development of this
B. 1,5 l                                            desease was caused by deficiency of the
C. 2,5 l                                            following compound:
D. 2,0 l                                            A. Naphtaquinone
E. 5,1                                              B. Retinol
83. In course of an experiment a toad's right       C. Biotin
labyrinth was destroyed. It will cause              D. 1,25 [OH]-dichydroxycholecalciferol
amyotonia of the following muscles:                 E. Tocopherol
A. Right and left extensors                         89. Histological examination of a skin tissue
B. Right extensors                                  sampling revealed granulomas consisting
C. Left flexors                                     of macrophagal nodules with lymphocytes
D. Right flexors                                    and plasmatic cells. There are also some big
E. Left extensors                                   macrophages with fatty vacuoles containing
84. A patient died from cardiopulmonary             causative agents of a disease packed up in
decompensation. Histological examination            form of spheres (Virchow's cells). Granulation
revealed diffused pulmonary lesion together         tissue is well vascularized. What disease
with interstitial edema. infiltration of tissue     is this granuloma typical for?
by limphocytes, macrophages, plasmocytes,           A. Rhinoscleroma
pulmonary fibrosis, panacinar emphysema.            B. Syphilis
What disease corresponds with the described         C. Lepra
picture?                                            D. Tuberculosis
A. Pulmonary atelectasis                            E. Glanders
B. Bronchial asthma                                 90. A patient consulted a stomatologist
C. Fibrosing alveolitis                             about purulent inflammation of his gums.
D. Chronic bronchitis                               What drug will be the most effective if it
E. Bronchopneumonia                                 is suspected that a causative agent is an
85. In order to accelerate healing of a radiation   anaerobe?
ulcer a vitamin drug was administered.              A. Oxacillin sodium
What drug is it?                                    B. Gentamicin
A. Methyluracil                                     C. Nitroxoline
D. Co-trimoxazole                                  diagnosis?
E. Metronidazole                                   A. Pediculosis
91. Microscopy of stained (Ziehl-Neelsen           B. Myiasis
staining) smears taken from the sputum             C. Scabies
of a patient with chronic pulmonary disease        D. Demodicosis
revealed red bacilli. What property                E. Phthiriasis
of tuberculous bacillus was shown up?              96. Examination of a patient with pustular
A. Acid resistance                                 skin lesions allowed to isolate a causative
B. Alcohol resistance                              agent that forms in the blood agar roundish
C. Capsule formation                               yellow middle-sized colonies surrounded by
D. Sporification                                   haemolysis zone. Smears from the colonies
E. Alkali resistance                               contain irregular-shaped clusters of grampositive
92. The upper lobe of the right lung               cocci. The culture is oxidase- and
is enlarged, grey and airless, the inscision       catalase-positive, ferments mannitol and
surface is dripping with turbid liquid,            synthesizes plasmocoagulase. What causative
the pleura has many fibrinogenous films;           agent was isolated?
microscopical examination of alveoles              A. Staphylococcus saprophyticus
revealed exudate containing neutrophils,           B. Staphylococcus aureus
desquamated alveolocytes and fibrin fibers.        C. Staphylococcus epidermidis
The bronchus wall is intact. What is the most      D. Streptococcus agalactiae
probable diagnosis?                                E. Streptococcus pyogenes
A. Focal pneumonia                                 97. Inoculum from pharynx of a patient
B. Pulmonary abscess                               ill with angina was inoculated into bloodtellurite
C. Inf1uenzalpneumonia                             agar. It resulted in growth of grey, radially
D. Croupous pneumonia                              striated (in form of rosettes) colonies 4-5 -mm in
E. Interstitial pneumonia                          dian1eter. Gram-positive bacilli with clublike
93. A patient has a decreased vasopressin          thickenings on their ends
synthesis that causes polyuria and as a result     placed in form of spread wide apart fingers
of it evident organism dehvdratation. What         are visible by microscope. What microorganisms
is the mechanism of polyuria development?          are these?
A. Reduced tubular reabsorption of protein         A. Streptococci
B. Reduced tubular reabsorption of Na ions         B. Botulism clostridia
C. Reduced glucose reabsorption                    C. Diphtheria corynebacteria
D. Reduced tubular reabsorption of water           D. Streptobacilli
E. Acceleration of glomerular filtration           E. Diphtheroids
94. A patient of surgical department               98. A man with a wound of his limb that had
complains about pain in the small of her           been suppurating for a long time died from
back and in the lower part of her belly; painful   intioxication. Autopsy revealed extreme
and frequent urination. Bacteriological            emaciation, dehydration, brown atrophy of
examination of urine revealed gramnegative         liver. myocardium, spleen and cross-striated
oxidase-positive rod-like bacteria                 muscles as well as renal amyloidosis. What
forming greenish mucoid colonies with. specific    diagnosis corresponds with the described picture?
smell. What causative agent can it be?             A. Chroniosepsis
A. Mycoplasma pneumonie                            B. Brucellosis
B. E.coli                                          C. Chernogubov's syndrome
C. Pseudomonas aeruginosa                          D. Septicopyemia
D. Proteus mirabilis                               E. Septicemia
E. Str.pyogenes                                    99. In course of severe respiratory viral
95. A patient has acne on his face. Microspcopic   infection there appeared clinical signs of
examination of scrapings from                      progressing cardiac insufficiency that caused
the affected areas revealed living porrect         death of a patient in the 2nd week of disease.
vermiform arthropoda 0,2-0,5 mm large with         Autopsy revealed that the heart was
four pairs of short extremities in the front       sluggish, with significant cavity dilatation.
part of their bodies. What is the laboratory       Histological examination of myocardium
revealed plephora of microvessels and diffuse      D. Sodium
infiltration of stroma by lymphocytes              E. Calcium
and histiocytes. What disease corresponds          104. An unconscious young man with signs
with the described picture?                        of morphine poisoning entered admission
A. Stenocardia                                     office. His respiration is shallow and
B. Cardiomyopathy                                  infrequent which is caused by inhibition of
C. Acute coronary insufficiency                    respiratory centre. What type of respiratory
D. Myocardium infarction                           failure is it?
E. Myocarditis                                     A. Ventilative restrictive
100. A patient consulted a doctor about            B. Perfusive
symmetric dermatitis of open skin areas. It        C. Ventilative obstructive
was found out that the patient lived mostly        D. Ventilative dysregulatory
on cereals and ate too little meat, milk and       E. Diffusive
eggs. What vitamin deficiency is the most          105. A patient being treated for viral
evident?                                           hepatitis type B got symptoms of hepatic
A. Nicotinamide                                    insufficiency. What blood changes indicative
B. Biotin                                          of protein metabolism disorder will be
C. Tocopherol                                      observed in this case?
D. Folic acid                                      A. Absolute hyperfibrinogenemia
E. Calciferol                                      B. Absolute hyperalbuminemia
101. A patient with clinical signs of              C. Absolute hypoalbuminemia
encephalitis was delivered to the infectious       D. Proteinic blood composition is unchanged
diseases hospital. Anamnesis registers a tick      E. Absolute hyperglobulinemia
bite. Hemagglutination-inhibition reaction         106. A patient ill with essential arterial
helped to reveal antibodies to the causative       hypertension had a hypertensic crisis that
agent of tick-borne encephalitis in the dilution   resulted in an attack of cardiac asthma.
1:20 which is not diagnostic. What actions         What is the leading mechanism of cardiac
should the doctor take after he had got such       ins insufficiency in this case?
result?                                            A. Absolute coronary insufficiency
A. To repeat examination with another              B. Heart overload caused by increased blood
diagnosticum                                       volume
B. To repeat the examination with scrum            C. Heart overload caused by high pressure
taken 10 days later                                D. Myocardium damage
C. To deny diagnosis of tick-borne encephalitis    E. Blood supply disturbance
D. To examine the same serum                       107. Surgical removal of a part of stomach
E. To apply more sensitive reaction                resulted in disturbed absorption of vitamin
102. Parents of a 10 year old boy consulted        B12 , it is excreted with feces. The patient
a doctor about extension of hair-covering,         was diagnosed with anemia. What factor is
growth of beard and moustache, low voice.          necessary for absorption of this vitamin?
Intensified secretion of which hormone must        A. Gastromucoprotein
be assumed?                                        B. Hydrochloric acid
A. Of testosterone                                 C. Gastrin
B. Of somatotropin                                 D. Folic acid
C. Of progesterone                                 E. Pepsin
D. Of oestrogen                                    108. A 45 year old man consulted a doctor
E. Of cortisol                                     about a plaque-like formation on his neck.
103. An animal with aortic valve insufficiency     Histological examination of a skin bioptate
got hypertrophy of its left heart ventricle.       revealed clusters of round and oval tumour
Some of its parts have local contractures.         cells with a narrow border of basophilic
What substance accumulated in the                  cytoplasm resembling of cells of basal epidermal
myocardiocytes caused these contractures?          layer. What tumour is it?
A. Lactic acid                                     A. Epidermal cancer
B. Potassium                                       B. Hydroadenoma
C. Carbon dioxide                                  C. Trichoepithelioma
D. Basal cell carcinoma                             114. The territory of an old burial ground for
E. Syringoadenoma                                   animal refuse that hasn't been used for over
109. A 45 year old woman is ill with breast         50 years is meant for house building. But soil
cancer. Her left arm has symptoms of                investigation showed the presense of viable
lymphatic system insufficiency - limb edema,        spores of a causative agent causing a very
lymph node enlargement. What form of                dangerous disease. What microorganism might
lymphatic circulation insufficiency is it?          have been preserved in soil for such a
A. Mechanic insufficiency                           long period of time?
B. -                                                A. Bacillus anthracis
C. Dynamic insufficiency                            B. Francisella tularensis
D. Combined insufficiency                           C. Mycobacterium bovis
E. Resorption insufficiency                         D. Brucella abortus
110. Mother of a 2 year old child consulted         E. Yersinia pestis
a stomatologist. In the period of pregnancy         115. During starvation muscle proteins
she was irregularly taking antibiotics for an       break up into free amino acids. These
infectious disease. Examination of the child        compounds will be the most probably
revealed incisor destruction, yellow enamel,        involved into the following process:
brown rim around the dental cervix. What            A. Glycogenolysis
drug has apparent teratogenic effect?               B. Decarboxylation
A. Ampiox                                           C. Gluconeogenesis in muscles
B. Octadine                                         D. Gluconeogenesis in liver
C. Furosemide                                       E. Synthesis of higher fatty acids
D. Doxacycline                                      116. A 62 year woman complains of frequent
E. Xantinol nicotinate                              pain attacks in the area of her chest
111. A man who has been taking a drug for           and backbone, rib fractures. Her doctor
a long time cannot withhold it because this         suspected myeloma (plasmocytoma). What
causes impairment of psychic, somatic and           of the following laboratory characteristics
vegetative functions. Name the syndrome             will be of the greatest diagnostic
of different disturbances caused by drug            importance?
discontinuation: A. Idiosyncrasy                    A. Hypoproteinemia
B. Sensibilization                                  B. Hyperalbuminemia
C. Tachyphylaxis                                    C. Hypoglobulinemia
D. Abstinence                                       D. Paraproteinemia
E. Cumulation                                       E. Proteinuria
112. A chemical burn caused esophagus               117. The cerebrospinal fluid is being examined
stenosis. Difficulty of ingestion led to the        for the purpose of diffrential meningitis
abrupt loss of weight. In blood: 3,0 * 1012/l,      diagnostics. At what site is the lumbal
Hb - 106 g/,. crude protein - 57 g/l. What          puncture safe?
type of starvation is it?                           A. L II - L III
A. Water                                            B. L V – S I
B. Incomplete                                       C. Th XIl - L I
C. Complete                                         D. L III – L IV
D. Proteinic                                        E. L I - L II
E. Absolute                                         118. Power inputs of a man were measured.
113. As a result of long-term starvation            In what state was this man if his power
the glomerular filtration of a man was              inputs were lower than basal metabolism?
accelerated by 20%. The most probable               A. Nervous tension
cause of filtration changes under such conditions   B. Sleep
is:                                                 C. Rest
A. Fall of oncotic pressure of blood plasma         D. Relaxation
B. Increase of renal plasma flow                    E. Simple work
C. Growth of filtration coefficient                 119. Vagi of an experimental animal were
D. Rise of systemic arterial pressure               cut on both sides. What respiration changes
E. Increased permeability of renal filter           will be observed?
A. It will become shallow and frequent            C. Heterotransplantation
B. It will become shallow and infrequent          D. Isotransplantation
C. No changes will be observed                    E. Allotransplantation
D. It will become deep and infrequent             125. A 40 year old man noticed a reddening
E. It will become deep and frequent               and an edema of skin in the area of his neck
120. Pharmacological effects of antidepressants   that later developed into a small abscess.
arc connected with inhibition                     The incised focus is dense, yellowish-green.
of an enzyme catalyzing biogenic amines           The pus contains white granules. Histological
noradrenaline and serotonine in the               examination revealed drusen of a fungus,
mitochondrions                                    plasmatic and xanthome cells, macrophages.
of cerebral neurons. What                         What type of mycosis is the most probable?
enzyme participates in this process?              A. Sporotrichosis
A. Lyase                                          B. Actinomycosis
B. Monoamine oxidase                              C. Aspergillosis
C. Transaminase                                   D. Candidosis
D. Peptidase                                      E. Coccidioidomycosis
E. Decarboxylase                                  126. An embryo displays disturbed process
121. A pathological process in bronchi            of dorsal mesoderm segmentation and somite
resulted in epithelium desquamation. What         formation. What part of skin will have
cells will regenerate bronchial epithelium?       developmental abnormalities?
A. Intercalary                                    A. Epidermis
B. Ciliate                                        B. Derma
C. Goblet                                         C. Sudoriferous glands
D. Basal                                          D. Sebaceous glands
E. Endocrinal                                     E. Hair
122. Examination of a man who had been            127. In course of a conditional experiment
working hard under higher temperature             the development of mesenchyma cells was
of the environment revealed abnormal              completely inhibited. Development of the
quantity of blood plasma proteins. What           following muscular tissue will be disturbed:
phenomenon is the case?                           A. Cardiac muscular tissue
A. Absolute hyperproteinemia                      B. Smooth muscular tissue
B. Paraproteinemia                                C. Skeletal muscular tissue
C. Relative hyperproteinemia                      D. Neural muscular tissue
D. Absolute hypoproteinemia                       E. Epidermal muscular tissue
E. Dysproteinemia                                 128. A girl is diagnosed with adrenogenital
123. A 70 year old man IS ill with                syndrome (pseudohermaphroditism). This
vascular atherosclerosis of lower extremities     pathology was caused by hypersecretion of
and coronary heart disease. Exan1ination          the following adrenal hormone:
revealed disturbance of lipidic                   A. Adrenalin
blood composition. The main factor of             B. Estrogen
atherosclerosis pathogenesis is the excess        C. Aldosterone
of the following lipoproteins:                    D. Androgen
A. Low-density lipoproteins                       E. Cortisol
B. Cholesterol                                    129. Golgi complex exports substances from
C. Chylomicrons                                   a cell due to the fusion of the membrane
D. High-density lipoproteins                      saccule with the cell membrane. The saccule
E. Intermediate density lipoproteins              contents flows out. What process is it?
124. For the purpose of myocardium                A. Exocytosis
infarction treatment a patient was injected       B. Facilitated diffusion
with embryonal stem cells derived from this       C. Endocytosis
very patient by means of therapeutic cloning.     D. Active- transport
What transplantation type is it?                  E. All answers arc false
A. Autotransplantation                            130. An oncological patient was prescribed
B. Xenotransplantation                            methotrexate. With the lapse of time target
cells of the tumour lost susceptibility to this     face, hirsutism, AP is 165/100 mm Hg, height
drug. There is change of gene expression of         - 164 cm, weight - 103 kg; the fat is
the folowing enzyme:                                mostly accumulated on her neck, thoracic
A. Thiaminase                                       girdle, belly. What is the main pathogenetic
B. Deaminase                                        mechanism of obesity?
C. Dehydrofolate reductase                          A. Increased production of glucocorticoids
D. Folate decarboxylase                             B. Reduced glucagon production
E. Folate oxidase                                   C. Increased insulin production
131. An infant has apparent diarrhea resulting      D. Reduced production of thyroid hormones
from improper feeding. One of the main              E. Increased mineralocorticoid production
diarrhea effects is plentiful excretion of sodium   136. A 20 year old patient died from
bicarbonate. What form of acid-base                 intoxication 8 days after artificial illegal
balance disorder is the case?                       abortion performed in her 14-15th week of
A. Metabolic alkalosis                              pregnancy. Autopsy of the corpse revealed
B. Respiratory alkalosis                            yellowish colour of eye sclera and of skin,
C. No disorders of acid-base balance will be        necrotic suppurative endometritis, multiple
observed                                            pulmonary abscesses, spleen hyperplasia
D. Respiratory acidosis                             with a big number of neutrophils in its sinuses.
E. Metabolic acidosis                               What complication after abortion was
112. Workers of a hothouse farm work under          developed?
conditions of unfavourable microclimate: air        A. Chroniosepsis
temperature is +37°C relative humidity is           B. Hemorrhagic shock
90%, air speed is 0,2 m/s. The way of heat          C. Septicopyemia
emission under these conditions will be:            D. Septicemia
A. Radiation                                        E. Viral hepatitis type A
B. Convection                                       137. There are several groups of molecular
C. Heat conduction                                  mechanisms playing important part in
D. All the ways                                     pathogenesis of insult to cells which contributes
E. Evaporation                                      to the pathology development. What
133. Examination of a 70 year old patient           processes are stimulated by proteinic
rrevealed insulin-dependent diabetes. What drug     damage mechanisms?
should be administered?                             A. Phospholipase activation
A. Glibenclamid                                     B. Osmotic membrane distension
B. Parathyroidin                                    C. Enzyme inhibition
C. Insulin                                          D. Lipid peroxidation
D. Mercazolilum                                     E. Acidosis
E. Cortisone                                        138. A patient takes digoxin for treatment
134. A 59 year old patient is a plant manager.      of cardiac insufficiency. What diuretic may
After the tax inspection of his plant he felt       increase digoxin toxicity due to the intensified
intense pain behind his breastbone irradiating      excretion of K+ ions?
to his left arm. 15 minutes later his condition     A. Lisinopril
came to normal. Which of the possible               B. Panangine
mechanisms of stenocardia development is            C. Siliborum
the leading in this case?                           D. Spironolactone
A. High catecholamine concentration m               E. Hydrochlorothiazide
blood                                               139. A patient ill with amebiasis was prescribed
B. Coronary atherosclerosis                         a certain drug. The use of alcohol
C. Functional heart overload                        together with this drug is contra-indicated
D. Coronary thrombosis                              because the drug inhibits metabolism of
E. Intravascular aggregation of blood               ethyl alcohol. What drug is it?
corpuscles                                          A. Clonidine
135. A 44 year old woman complains of               B. Metronidazole
general weakness, heart pain, significant           C. Reserpine
increase of body weight. Objectively: moon          D. Diazepam
E. Aminazine                                      dysentery and was once more infected with
140. While performing an operation in the         the same causative agent. What is such
area of axillary crease a surgeon has to define   infection form called?
an arterial vessel surrounded by fascicles        A. Persisting infection
of brachial plexus. What artery is it?            B. Chronic infection
A. A.axillaris                                    C. Recidivation
B. A.profunda brachii                             D. Superinfection
C. A. vertebralis                                 E. Reinfection
D. A.subscapularis                                146. A 30 year old woman has applied a lipstick
E. A. transversa colli                            with a fluorescent substance for a
141. A patient with nephrotic syndrome has        long time. Then she got a limited erythema
massive edemata of his face and limbs. What       and slight peeling on her lip border, later
is the leading pathogenelic mechanism of          there appeared transversal striae and cracks.
edemata development?                              Special methods of microscopic examination
A. Rise of hydrodynamic blood pressure            of the affected area helped to reveal
B. Lymphostasis                                   sensibilized lymphocytes and macrophages
C. Increase of vascular permeability              in the connective tissue: cytolysis. What
D. Increase of lymph outflow                      type of immunological hypersensitivity was
E. Drop of oncotic blood pressure                 developed?
142. A patient underwent an operation on          A. III type (immune complex cytotoxicity)
account of gall bladder excision that resulted    B. I type (reaginic)
in obstruction of Ca absorption through the       C. IVtype (cellular cytotoxicity)
bowels wall. What vitamin will stimulate this     D. II type (antihody cytotoxicity)
process?                                          E. Granulomatosis
A.C                                               147. Autopsy of a man who died from
B. B12                                            the sepsis in his femoral hone revealed
C. D3                                             phlegmonous inflammation that affected the
D. K                                              marrow, haversian canals and periosteum.
E. PP                                             Under the periosteum there are multiple
143. While studying maximally spiralized          abscesses, adjoining soft tissues of thigh also
chromosomes of human karyotype the                have signs of phlegmonous inflammation.
process of cell division was stopped in the       What pathological process was described?
following phase:                                  A. Chronic henlatogenous osteomielitis
A. Telophase                                      B. Osteoporosis
B. Interphase                                      c. -
C. Anaphase                                       D. Acute hemalogenous osteomyelitis
D. Metaphase                                      E. Osteopetrosis
E. Prophase                                       148. A patient was diagnosed with autoimmune
144. A patient ill with tuberculosis died         hemolitic cytotoxic anemia. What
from progressing cardiopulmonary                  substances are antigens in II type allergic
decompensation. Autopsy in the area of            reactions?
the right lung apex revealed a cavity 5           A. Antibiotics
cm in diameter communicating with lumen           B. Hormones
of a segmental bronchus. On the inside            C. Serum proteins
cavity walls are covered with caseous             D. Modified receptors of cell membranes
masses with epithelioid and Langhans cells        E. Inflammation modulators
beneath them. What morphological form of          149. Rats being under stress have muscular
tuberculosis is it?                               hypertonia and high arterial pressure, high
A. Acute focal tuberculosis                       glucose concentration in blood and intensified
B. Infiltrative tuberculosis                      secretion of corticotropin and corticosteroids.
C. Caseous pneumonia                              In what stress phase are these animals?
D. Acute cavernous tuberculosis                   A. Exhaustion
E. Tuberculoma                                    B. Terminal
145. A patient recovered from Sonne               C. Antishock phase
D. Erectile                                        155. A 38 year old patient with full-blown
E. Shock phase                                     jaundice, small cutaneous hemorrhages,
150. It was proved that a molecule of              general weakness and loss of appetite
immature mRNA (precursor mRNA)                     underwent puncture biopsy of liver. Histological
contained more triplets than amino acids           examination revealed disseminated
found in the synthesized protein. The              dystrophy, hepatocyte necrosis,
reason for that is that translation is normally    Councilman's bodies. Lobule periphery
preceded by:                                       has signs of significant infiltration by
A. Processing                                      lymphocytes, there are also individual
B. Initiation                                      multinuclear
C. Reparation                                      hepatocytes. What is the most
D. Replication                                     probable diagnosis?
E. Mutation                                        A. Acute alcoholic hepatitis
151. A patient who has been treated in a           B. Toxic degeneration of liver
neural clinic and has been taking a sedative       C. Acute viral hepatitis
for a long time got the following complication:    D. Chronic hepatitis
cough, rhinitis, epiphora. What drug               E. Miliary hepatic cirrhosis
caused these disturbances?                         156. A patient with acute morphine poisoning
A. Sodium bromide                                  was delivered to a hospital. What specific
B. Valerian                                        narcotic antagonist should be chosen in this
C. Phenazepam                                      case?
D. Reserpine                                       A. Unithiol
E. Diazepam                                        B. Paracetamol
152. Examination of a newborn boy's                C. Digoxin
genitalia revealed an urethral hiatus that         D. Methacin
opens on the undersite of his penis. What          E. Naloxone
malformation is it?                                157. Autopsy of a man ill with severe
A. Cryptorhidism                                   hypothyroidism revealed that connective tissue,
B. Epispadia                                       organ stroma, adipose and cartilaginous
C. Hermaphroditism                                 tissues were swollen, semitransparent,
D. Monorchism                                      mucus-like. Microscopic examination of tissues
E. Hypospadias                                     revealed stellate cells having processes
153. A patient that entered the admission office   with mucus between them. What type of
had the following signs of acute cardiac           dystrophy is it?
insuffiency: paleness, acrocyanosis, frequent      A. Stromal-vascular carbohydrate
shallow respiration. What drug is                  B. Strornal-vascular proteinaceous
indicated in this case?                            C. Parenchymatous adipose
A. Corglycon                                       D. Parenchymatous proteinaceous
B. Adrenaline hydrochloride                        E. Stromal-vascular adipose
C. Nitroglycerine                                  158. In order to estimate toxigenity of diphtheria
D. Cordiamin                                       agents obtained from patients the
E. Digitoxin                                       cultures were inoculated on Petri dish with
154. In order to speed up healing of a wound       nutrient agar on either side of a filter paper
of oral mucosa a patient was prescribed a          strip that was put into the centre and moistened
drug that is a thermostable protein occuring       with antidiphtheric antitoxic serum.
in tears, saliva, mother's milk as well as in      After incubation of inoculations in agar the
a new-laid hen's egg. It is known that this        strip-like areas of medium turbidity were
protein is a factor of natural resistance of an    found between separate cultures and the
organism. What is it called?                       strip of filter paper. What immunological
A. Complement                                      reaction was conducted?
B. Imanine                                         A. Rings precipitation reaction
C. lnterleukin                                     B. Agglutination reaction
D. Interferon                                      C. Precipitation gel reaction
E. Lysozyme                                        D. Coomb's test
E. Opsonization reaction                          caused complete disruption of spinal cord at
159. A patient ill with enteritis accompanied     the level of the first cervical segment. What
by massive diarrhea has low water rate in         respiration changes will be observed?
the extracellular space, high water rate inside   A. No changes will be observed
the cells and low blood osmolarity. What          B. It will become infrequent and deep
is such disturbance of water-electrolytic         C. Thoracic respiration will be maintained,
metabolism called?                                diaphragmal respiration will disappear
A. Hyperosmolar hypohydration                     D. It will come to a standstill
B. Hyperosmolar hyperhydration                    E. Diaphragmal respiration will be maintained,
C. Osmolar hypohydration                          thoracic respiration will disappear
D. Hypo-osmolar hyperhydration                    165. Systemic arterial pressure of an adult
E. Hypo-osmolar hypohydration                     dropped from 120/70 to 90/50 mm Hg
160. In course of a preventive examination        that led to reflectory vasoconstriction. The
of a miner a doctor revealed changes of           vasoconstriction will be maximal in the
cardiovascular fitness which was indicative       following organ:
of cardiac insufficiency at the compensation      A. Bowels
stage. What is the main proof of cardiac          B. Heart
compensation?                                     C. Kidneys
A. Myocardium hypertrophy                         D. Adrenals
B. Tachycardia                                    E. Brain
C. Cyanosis                                       166. A pregnant woman was registered in
D. Dyspnea                                        an antenatal clinic and underwent complex
E. Rise of arterial pressure                      examination for a number of infections.
161. It was found out that some compounds,        Blood serum contained IgM to the rubella
for instance fungi toxins and some antibiotics    virus. What is this result indicative of?
can inhibit activity of RNA-polymerase.           A. Of exacerbation of a chronic disease
What process will be disturbed in a cell in       B. Of recurring infection with rubella virus
case of inhibition of this enzyme?                C. Of a chronic process
A. Reparation                                     D. The woman is healthy
B. Translation                                    E. Of primary infection
C. Transcription                                  167. A boy found a spider with the following
D. Replication                                    morphological characteristics: it is 2 cm
E. Processing                                     long, has roundish black abdomen with two
162. As a result of destruction of certain        rows of red spots on its dorsal side; four pairs
brainstem structures an animal has lost           of jointed limbs are covered with small
its orientative reflexes in response to           black hairs. What arthropod is it?
strong photic stimuli. What structures were       A. Mite
destroyed?                                        B. Solpuga
A. Posterior tubercles of quadrigeminal plate     C. Karakurt spider
B. Anterior tubercles of quadrigeminal plate      D. Scorpion
C. Red nuclei                                     E. Tarantula
D. Black substance                                168. Examination of a young woman
E. Vestibular nuclei                              revealed a node-like, soft and elastic
163. Inflammation of the tympanic cavity          homogenous tumour of pinkish-white
(purulent otitis media) was complicated           colour along the acoustic nerve. The tumour
by inflammation of mammillary process             contains cell bundles with oval nuclei.
sockets. What wall of tympanic cavity did         Cellular fibrous bundles form rhythmic
the pus penetrate into the sockets through?       structures made up by parallel rows of
A. Lateral                                        regularly oriented cells arranged in form
B. Medial                                         of a palisade with cell-free homogenous
C. Anterior                                       zone (Verocay bodies) between them. What
D. Posterior                                      tumour is it?
E. Superior                                       A. Neurinoma
164. A 35 year old man got an injury that         B. Ganglioneuroblastoma
C. Malignant neurinoma                            chromophilic and chromophobic. Identify
D. Neuroblastoma                                  this organ:
E. Ganglioneurinoma                               A. Hypothalamus
169. Two hours after an exam a student had        B. Adrenal glands
a blood count done and it was revealed            C. Hypophysis
that he had leukocytosis without significant      D. Thyroid gland
leukogram modifications. What is the              E. Parathyroid gland
most probable mechanism of leukocytosis           174. A patient came to the hospital complaining
development?                                      about quick fatigability and apparent
A. Redistribution of leukocytes in the organism   muscle weakness. Examination revealed
B. Deceleration of leukocyte migration to         an autoimmune disease that causes disorder
the tissues                                       of functional receptor condition in
C. Leukopoiesis intensification and               neuromuscular synapses. What transmitter
deceleration of leukocyte lysis                   will be blocked?
D. Deceleration of leukocyte lysis                A. Noradrenalin
E. Leukopoiesis intensification                   B. Serotonin
170. If a man has an attack of bronchiospasm      C. Glyeine
it is necessary to reduce the effect of           D. Dopamine
vagus on smooth muscles of bronchi. What          E. Acetylcholine
membrane cytoreceptors should be blocked          175. Microscopic examination of a Gramstained
for this purpose?                                 scrape from patient's tongue
A. M-cholinoreceptors                             revealed oval, round, elongated chains of
B. β-adrenoreceptors                              dark-violet gemmating cells. What disease
C. α- and β-adrenoreceptors                       can be caused by this causative agent?
D. α-adrenoreceptors                              A. Staphylococcic infection
E. .N-cholinoreceptors                            B. Streptococcic infection
171. Plasmic factors of blood coagulation         C. Diphtheria
are exposed to post-translational modification    D. Actinomycosis
with the participation of vitamin K. It           E. Candidosis
is necessary as a cofactor in the enzyme          176. A patient ill with chronic gastritis went
system of γ-carboxylation of protein factors      for endogastric pH-metry that allowed to
cif blood coagulation due to the increased        reveal decreased acidity of gastric juice. It
affinity of their molecules with calcium ions.    is indicative of diminished function of the
What amino acid is carboxylated in these          following cells:
proteins?                                         A. Parietal exocrinocytes
A. Valine                                         B. Accessory cells
B. Arginine                                       C. Endocrinocytes
C. Phenylalanine                                  D. Cervical cells
D. Glutamic                                       E. Chief exocrinocytes
E. Serine                                         177. Examination of a 60 year old patient
172. A sensitive neural ganglion consists of      revealed hyperglycemia and glucosuria. A
roundish neurocytes with one extension that       doctor administered him a medication for
divides into axon and dendrite at some distance   internal use. What medication is it?
from the perikaryon. What are these               A. Pancreatine
cells called?                                     B. Furosemide
A. Multipolar                                     C. Oxytocin
B. Pseudounipolar                                 D. Glibenclamid
C. Unipolar                                       E. Corglycon
D. Apolar                                         178. A 6 month old baby ill with bronchitis
E. Bipolar                                        was taken for an X-ray of chest. Apart
173. An endocrinal gland with parenchyma          of changes associated with bronchi the X-ray
consisting of epithelium and neural tissue        film showed a shadow of thymus gland.
is under morphological examination. Epithelial    What might have caused such changes?
trabecules have two types of cells:               A. It is caused by thymus inflammation
B. It is caused by abnormal position               rhinitis and loss of sense of smell for a week.
C. It is caused by neoplastic process              Objectively: nasal cavity contains a lot of
D. The above-mentioned condition is a              mucus that covers mucous membrane and
normal variant for this age                        blocks olfactory receptors. In what part of
E. It's the effect of bronchitis                   nasal cavity are these receptors situated?
179. A 20 year old patient complains of            A. Common nasal meatus
general weakness, dizziness, quick fatigability.   B. Vestibule of nose
Blood analysis results: Hb- 80                     C Median nasal turbinate
g/l. Microscopical examination results:            D. Inferior nasal turbinate
erythrocytes are of modified form. This            E. Superior nasal turbinate
condition might be caused by:                      185. A cardiac electric stimulator was
A. Obturative jaundice                             implanted to a 75 year old man with heart
B. Acute intermittent porphyria                    rate of 40 bpm. Thereafter the heart rate
C. Sickle-cell anemia                              rose up to 70 bpm. The electric stimulator
D. Addison's disease                               has undertaken the function of the following
E. Hepatocellular jaundice                         heart part:
180. In course of laparotomy a surgeon             A. His' bundle branches
revealed gangrenous lesion of descending           B. Sinoatrial node
colon. It was caused by thrombosis of the          C. His' bundle fibel's
following artery:                                  D. Purkinje's fibers
A. Dexter colic                                    E. Atrioventricular node
B. Median colic                                    186. Blood of a patient with presumable
C. Ileocolic                                       sepsis was inoculated into sugar broth.
D. Sinister colic                                  There appeared bottom sediment. Repeated
E. Superior mesenteric artery                      inoculation into blood agar caused growth
181. In course of a small pelvis operation it      of small transparent round colonies
became necessary to ligate an ovarian artery.      surrounded by hemolysis zone. Examination
What formation may be accidentally ligated         of a smear from the sediment revealed
together with it?                                  gram-positive cocci in form of long chains.
A. Round ligament of uterus                        What microorganisms are present in blood
B. Urethra ~                                       of this patient?
C. Uterine tube                                    A. Streptococci
D. Internal iliac vein                             B. Tetracocci
E. Ureter                                          C. Sarcina
182. A patient has a deep cut wound on the         D. Staphylococci
posterior surface of his shoulder in its middle    E. Micrococci
third. What muscle might he injured?               187. A patient has low rate of magnesium
A. Anconeus muscle                                 ions that are necessary for affixion of ribosomes
B. Biceps muscle of arm                            to the endoplasmic reticulum. It is
C Brachial muscle                                  known that it causes disturbance of protein
D. Coracohrachial muscle                           biosynthesis. At what stage is protein
E. Triceps muscle of arm                           biosynthesis
183. A patient was admitted to the surgical        impaired?
department with suspected inflammation of          A. Termination
Meckel`s diverticulum. What part of bowels         B. Translation
should be examined in order to discover the        C. Amino acid activation
diverticulum in course of an operation?            D. Replication
A. Ileum                                           E. Transcription
B. Jejunmum                                        188. A patient with hypertensic crisis was
C Colon ascendens                                  admitted to the cardiological department,
D. Duodenum                                        he was injected intravenously with an
E. Caecum                                          antihypertensive
184. A 35 year old patient applied to a            drug - salt of an alkaline-earth
doctor with complaints about having intense        metal. What drug was injected?
A. Benzohexamethonium                             C. Louis-Bar
B. Potassium chloride                             D. Wiskott-Aldrich
C Sodium hydrocarbonate                           E. DiGeorge
D. Magnesium sulfate                              195. A woman suffering from osteochondrosis
E. Caleium lactate                                felt acute pain in her humeral articulation
189. Examination of a 35 year old patient         that became stronger when she abducted her
revealed high acidity of gastric juice. What      shoulder. These symptoms might be caused
receptors should be blocked in order to           by damage of the following nerve:
reduce it?                                        A. Dorsal scapular nerve
A. α 1-adrenoreceptors                            B. Axillary nerve
B. β2-adrenoreceptors                             C. Subscapular nerve
C. β1 -adrenoreceptors                            D. Subclavicular nerve
D. α2-adrenoreceptors                             E. Throracodorsal nerve
E. Histamine                                      196. A boy has I (I0 I0) blood group and
190. While performing an inguinal canal           his sister has IV (IA IB) blood group. What
operation on account of hernia a surgeon          blood groups do their parents have?
damaged the canal's contents. What exactly        A. III (IB I0) and IV (IA IB)
was damaged?                                      B. II (IA IA) and III (IB I0)
A. Funiculus spermaticus                          C. II (IA I0) and III (IB I0)
B. Lig. teres uteri                               D. I (I0 I0) and III (IB I0)
C. -                                              E. I (I0 I0) and IV (IA IB)
D. Lig. inguinale                                 197. A patient has a cluster of matted
E. Urarchus                                       together dense lymph nodes on his neck.
191. A 3 year old child with fever was given      Histological
aspirin. It resulted in intensified erythrocyte   examination of a removed lymph
haemolysis. Hemolytic anemia might have           node revealed proliferation of reticular cells,
been caused by congenital insufficiency of        presense of Reed-Sternberg cells. What disease
the following enzyme:                             is meant?
A. Glycogen phosphorylase                         A. Myeloblastic leukosis
B. Glucose 6-phosphate dehydrogenase              B. Lyimphogranulomatosis
C. Glycerol phosphate dehydrogenase               C. Lymphoblastic leukosis
D. Glucose 6-phosphatase                          D. Myelocytic leukosis
E. γ-glutamiltransferase                          E. Lympholcytic leukosis
192. After consumption of rich food a patient     198. A woman underwent an operation on
has nausea and heartburn, steatorrhea.            account of extrauterine (tubal) pregnancy.
This condition might he caused by:                In course of the operation the surgeon
A. Increased lipase secretion                     should ligate the branches of the following
B. Bile acid deficiency                           arteries:
C. Amylase deficiency                             A. Uterine and superior cystic
D. Disturbed phospholipase synthesis              B. Superior cystic and ovarian
E. Disturbed tripsin synthesis                    C. Uterine and inferior cystic
193. A patient complains of dizziness and         D. Uterine and ovarian
 hearing loss. What nerve is damaged?             E. Inferior cystic and ovarian
A. Vestibulocochlear                              199. A specimen stained by Ozheshko
B. Vagus                                          method contains rod-like microorganisms
C. Sublingual                                     stained blue with round terminal
D. Trochlear                                      components stained red. What are these
E. Trigeminus                                     components called?
194. A child was born with cleft palate.          A. Capsules
Examination revealed aorta defects and            B. Cilia
reduced number of T-Iymphocytes in blood.         C. Mesosomas
What immunodeficient syndrome is it?              D. Spores
A. Swiss-type                                     E. Flagella
B. Chediak-Higashi                                200. A 37 year old patient suffering from
obliterating vascular endarteritis of lower
limbs takes daily 60 microgram/kilogram
of phenylin. Because of presentations
of convulsive disorder (craniocerebral
trauma in anamnesis) he was prescribed
phenobarbital. Withholding this drug
caused nasal hemorhage. What is this
complication connected with?
A. Aliphatic hydroxylation of phenobarbital
B. Inhibition of microsomal oxidation in liver
caused by phenobarbital
C. Induction of enzymes of microsomal
oxidation in liver caused by phenobarbital
D. Conjugation of phenylin with glucuronic
acid
E. Oxidative deamination of phenylin

More Related Content

What's hot

Krok 1 - 2014 (Microbiology)
Krok 1 -  2014 (Microbiology)Krok 1 -  2014 (Microbiology)
Krok 1 - 2014 (Microbiology)Eneutron
 
Krok 1 - 2009 Question Paper (General medicine)
Krok 1 - 2009 Question Paper (General medicine)Krok 1 - 2009 Question Paper (General medicine)
Krok 1 - 2009 Question Paper (General medicine)Eneutron
 
Krok 1 - 2010 Question Paper (General medicine)
Krok 1 - 2010 Question Paper (General medicine)Krok 1 - 2010 Question Paper (General medicine)
Krok 1 - 2010 Question Paper (General medicine)Eneutron
 
Krok 1 - 2015 (Microbiology)
Krok 1 - 2015 (Microbiology)Krok 1 - 2015 (Microbiology)
Krok 1 - 2015 (Microbiology)Eneutron
 
Krok 1 2014 - biochemistry
Krok 1   2014 - biochemistryKrok 1   2014 - biochemistry
Krok 1 2014 - biochemistryEneutron
 
Krok 1 - 2014 (Biochemistry)
Krok 1 - 2014 (Biochemistry)Krok 1 - 2014 (Biochemistry)
Krok 1 - 2014 (Biochemistry)Eneutron
 
Krok 1 - 2006 Question Paper (General medicine)
Krok 1 - 2006 Question Paper (General medicine)Krok 1 - 2006 Question Paper (General medicine)
Krok 1 - 2006 Question Paper (General medicine)Eneutron
 
Krok 1 - 2014 (Anatomy)
Krok 1 -  2014 (Anatomy)Krok 1 -  2014 (Anatomy)
Krok 1 - 2014 (Anatomy)Eneutron
 
Krok 1 - 2007 Question Paper (General medicine)
Krok 1 - 2007 Question Paper (General medicine)Krok 1 - 2007 Question Paper (General medicine)
Krok 1 - 2007 Question Paper (General medicine)Eneutron
 
Krok 1 - 2013 Question Paper (General medicine)
Krok 1 - 2013 Question Paper (General medicine)Krok 1 - 2013 Question Paper (General medicine)
Krok 1 - 2013 Question Paper (General medicine)Eneutron
 
Krok 1 - 2015 (Physiology)
Krok 1 - 2015 (Physiology)Krok 1 - 2015 (Physiology)
Krok 1 - 2015 (Physiology)Eneutron
 
Krok 1 - 2011 Question Paper (General medicine)
Krok 1 - 2011 Question Paper (General medicine)Krok 1 - 2011 Question Paper (General medicine)
Krok 1 - 2011 Question Paper (General medicine)Eneutron
 
Krok 1 - 2012 Question Paper (General medicine)
Krok 1 - 2012 Question Paper (General medicine)Krok 1 - 2012 Question Paper (General medicine)
Krok 1 - 2012 Question Paper (General medicine)Eneutron
 
Krok 1 - 2015 (Anatomy)
Krok 1 - 2015 (Anatomy)Krok 1 - 2015 (Anatomy)
Krok 1 - 2015 (Anatomy)Eneutron
 
Krok 1 - 2015 Base (General Medicine)
Krok 1 - 2015 Base (General Medicine)Krok 1 - 2015 Base (General Medicine)
Krok 1 - 2015 Base (General Medicine)E_neutron
 
Krok 1 - 2015 (Biology)
Krok 1 - 2015 (Biology)Krok 1 - 2015 (Biology)
Krok 1 - 2015 (Biology)Eneutron
 
Krok 1 - 2005 Question Paper (General medicine)
Krok 1 - 2005 Question Paper (General medicine)Krok 1 - 2005 Question Paper (General medicine)
Krok 1 - 2005 Question Paper (General medicine)Eneutron
 
Krok 1 Medicine - 2016 General Medicine
Krok 1 Medicine - 2016 General MedicineKrok 1 Medicine - 2016 General Medicine
Krok 1 Medicine - 2016 General MedicineEneutron
 
Krok 1 - 2015 (Path-Physiology)
Krok 1 - 2015 (Path-Physiology)Krok 1 - 2015 (Path-Physiology)
Krok 1 - 2015 (Path-Physiology)Eneutron
 

What's hot (20)

Krok 1 - 2014 (Microbiology)
Krok 1 -  2014 (Microbiology)Krok 1 -  2014 (Microbiology)
Krok 1 - 2014 (Microbiology)
 
Krok 1 - 2009 Question Paper (General medicine)
Krok 1 - 2009 Question Paper (General medicine)Krok 1 - 2009 Question Paper (General medicine)
Krok 1 - 2009 Question Paper (General medicine)
 
Krok 1 - 2010 Question Paper (General medicine)
Krok 1 - 2010 Question Paper (General medicine)Krok 1 - 2010 Question Paper (General medicine)
Krok 1 - 2010 Question Paper (General medicine)
 
Krok 1 - 2015 (Microbiology)
Krok 1 - 2015 (Microbiology)Krok 1 - 2015 (Microbiology)
Krok 1 - 2015 (Microbiology)
 
Krok 1 2014 - biochemistry
Krok 1   2014 - biochemistryKrok 1   2014 - biochemistry
Krok 1 2014 - biochemistry
 
Krok 1 - 2014 (Biochemistry)
Krok 1 - 2014 (Biochemistry)Krok 1 - 2014 (Biochemistry)
Krok 1 - 2014 (Biochemistry)
 
Krok 1 - 2006 Question Paper (General medicine)
Krok 1 - 2006 Question Paper (General medicine)Krok 1 - 2006 Question Paper (General medicine)
Krok 1 - 2006 Question Paper (General medicine)
 
Krok 1 - 2014 (Anatomy)
Krok 1 -  2014 (Anatomy)Krok 1 -  2014 (Anatomy)
Krok 1 - 2014 (Anatomy)
 
Krok 1 - 2007 Question Paper (General medicine)
Krok 1 - 2007 Question Paper (General medicine)Krok 1 - 2007 Question Paper (General medicine)
Krok 1 - 2007 Question Paper (General medicine)
 
2006
20062006
2006
 
Krok 1 - 2013 Question Paper (General medicine)
Krok 1 - 2013 Question Paper (General medicine)Krok 1 - 2013 Question Paper (General medicine)
Krok 1 - 2013 Question Paper (General medicine)
 
Krok 1 - 2015 (Physiology)
Krok 1 - 2015 (Physiology)Krok 1 - 2015 (Physiology)
Krok 1 - 2015 (Physiology)
 
Krok 1 - 2011 Question Paper (General medicine)
Krok 1 - 2011 Question Paper (General medicine)Krok 1 - 2011 Question Paper (General medicine)
Krok 1 - 2011 Question Paper (General medicine)
 
Krok 1 - 2012 Question Paper (General medicine)
Krok 1 - 2012 Question Paper (General medicine)Krok 1 - 2012 Question Paper (General medicine)
Krok 1 - 2012 Question Paper (General medicine)
 
Krok 1 - 2015 (Anatomy)
Krok 1 - 2015 (Anatomy)Krok 1 - 2015 (Anatomy)
Krok 1 - 2015 (Anatomy)
 
Krok 1 - 2015 Base (General Medicine)
Krok 1 - 2015 Base (General Medicine)Krok 1 - 2015 Base (General Medicine)
Krok 1 - 2015 Base (General Medicine)
 
Krok 1 - 2015 (Biology)
Krok 1 - 2015 (Biology)Krok 1 - 2015 (Biology)
Krok 1 - 2015 (Biology)
 
Krok 1 - 2005 Question Paper (General medicine)
Krok 1 - 2005 Question Paper (General medicine)Krok 1 - 2005 Question Paper (General medicine)
Krok 1 - 2005 Question Paper (General medicine)
 
Krok 1 Medicine - 2016 General Medicine
Krok 1 Medicine - 2016 General MedicineKrok 1 Medicine - 2016 General Medicine
Krok 1 Medicine - 2016 General Medicine
 
Krok 1 - 2015 (Path-Physiology)
Krok 1 - 2015 (Path-Physiology)Krok 1 - 2015 (Path-Physiology)
Krok 1 - 2015 (Path-Physiology)
 

Similar to 2008

Krok 1 - 2015 (Histology)
Krok 1 - 2015 (Histology)Krok 1 - 2015 (Histology)
Krok 1 - 2015 (Histology)Eneutron
 
Krok 1 - 2014 (Path-Physiology)
Krok 1 - 2014 (Path-Physiology)Krok 1 - 2014 (Path-Physiology)
Krok 1 - 2014 (Path-Physiology)Eneutron
 
Krok1 stomatology - 2016
Krok1   stomatology - 2016Krok1   stomatology - 2016
Krok1 stomatology - 2016Eneutron
 
Krok 1 - 2012 Question Paper (Stomatology)
Krok 1 - 2012 Question Paper (Stomatology)Krok 1 - 2012 Question Paper (Stomatology)
Krok 1 - 2012 Question Paper (Stomatology)Eneutron
 
Krok 1 - 2015 Base (General Medicine)
Krok 1 - 2015 Base (General Medicine)Krok 1 - 2015 Base (General Medicine)
Krok 1 - 2015 Base (General Medicine)Eneutron
 
Krok 1 - 2009 Question Paper (Stomatology)
Krok 1 - 2009 Question Paper (Stomatology)Krok 1 - 2009 Question Paper (Stomatology)
Krok 1 - 2009 Question Paper (Stomatology)Eneutron
 
Krok 1 - 2015 Question Paper (Pharmacy)
Krok 1 - 2015 Question Paper (Pharmacy)Krok 1 - 2015 Question Paper (Pharmacy)
Krok 1 - 2015 Question Paper (Pharmacy)Eneutron
 
Krok 2 - 2013 (Pediatrics)
Krok 2 - 2013 (Pediatrics)Krok 2 - 2013 (Pediatrics)
Krok 2 - 2013 (Pediatrics)Eneutron
 
Krok 1 - 2015 Question Paper (Stomatology)
Krok 1 - 2015 Question Paper (Stomatology)Krok 1 - 2015 Question Paper (Stomatology)
Krok 1 - 2015 Question Paper (Stomatology)Eneutron
 
03.sabiston surgery questions 17th ed
03.sabiston surgery questions 17th ed03.sabiston surgery questions 17th ed
03.sabiston surgery questions 17th edLucia Rosales
 
Krok 1 - 2007 Question Paper (Stomatology)
Krok 1 - 2007 Question Paper (Stomatology)Krok 1 - 2007 Question Paper (Stomatology)
Krok 1 - 2007 Question Paper (Stomatology)Eneutron
 
ImmunoFinal (1)-converted.pdf
ImmunoFinal (1)-converted.pdfImmunoFinal (1)-converted.pdf
ImmunoFinal (1)-converted.pdfJamesLTorres
 
STEP 2 - Nums past paper by dr. shahid alam
STEP 2 - Nums past paper by dr. shahid alamSTEP 2 - Nums past paper by dr. shahid alam
STEP 2 - Nums past paper by dr. shahid alamDr. Shadab Kamal
 
Krok 1 - 2006 Question Paper (Stomatology)
Krok 1 - 2006 Question Paper (Stomatology)Krok 1 - 2006 Question Paper (Stomatology)
Krok 1 - 2006 Question Paper (Stomatology)Eneutron
 
Krok 2 - 2014 (Pediatrics)
Krok 2 - 2014 (Pediatrics)Krok 2 - 2014 (Pediatrics)
Krok 2 - 2014 (Pediatrics)Eneutron
 

Similar to 2008 (19)

2007
20072007
2007
 
2005
20052005
2005
 
Krok 1 - 2015 (Histology)
Krok 1 - 2015 (Histology)Krok 1 - 2015 (Histology)
Krok 1 - 2015 (Histology)
 
2009
20092009
2009
 
Krok 1 - 2014 (Path-Physiology)
Krok 1 - 2014 (Path-Physiology)Krok 1 - 2014 (Path-Physiology)
Krok 1 - 2014 (Path-Physiology)
 
Krok1 stomatology - 2016
Krok1   stomatology - 2016Krok1   stomatology - 2016
Krok1 stomatology - 2016
 
Krok 1 - 2012 Question Paper (Stomatology)
Krok 1 - 2012 Question Paper (Stomatology)Krok 1 - 2012 Question Paper (Stomatology)
Krok 1 - 2012 Question Paper (Stomatology)
 
file
file file
file
 
Krok 1 - 2015 Base (General Medicine)
Krok 1 - 2015 Base (General Medicine)Krok 1 - 2015 Base (General Medicine)
Krok 1 - 2015 Base (General Medicine)
 
Krok 1 - 2009 Question Paper (Stomatology)
Krok 1 - 2009 Question Paper (Stomatology)Krok 1 - 2009 Question Paper (Stomatology)
Krok 1 - 2009 Question Paper (Stomatology)
 
Krok 1 - 2015 Question Paper (Pharmacy)
Krok 1 - 2015 Question Paper (Pharmacy)Krok 1 - 2015 Question Paper (Pharmacy)
Krok 1 - 2015 Question Paper (Pharmacy)
 
Krok 2 - 2013 (Pediatrics)
Krok 2 - 2013 (Pediatrics)Krok 2 - 2013 (Pediatrics)
Krok 2 - 2013 (Pediatrics)
 
Krok 1 - 2015 Question Paper (Stomatology)
Krok 1 - 2015 Question Paper (Stomatology)Krok 1 - 2015 Question Paper (Stomatology)
Krok 1 - 2015 Question Paper (Stomatology)
 
03.sabiston surgery questions 17th ed
03.sabiston surgery questions 17th ed03.sabiston surgery questions 17th ed
03.sabiston surgery questions 17th ed
 
Krok 1 - 2007 Question Paper (Stomatology)
Krok 1 - 2007 Question Paper (Stomatology)Krok 1 - 2007 Question Paper (Stomatology)
Krok 1 - 2007 Question Paper (Stomatology)
 
ImmunoFinal (1)-converted.pdf
ImmunoFinal (1)-converted.pdfImmunoFinal (1)-converted.pdf
ImmunoFinal (1)-converted.pdf
 
STEP 2 - Nums past paper by dr. shahid alam
STEP 2 - Nums past paper by dr. shahid alamSTEP 2 - Nums past paper by dr. shahid alam
STEP 2 - Nums past paper by dr. shahid alam
 
Krok 1 - 2006 Question Paper (Stomatology)
Krok 1 - 2006 Question Paper (Stomatology)Krok 1 - 2006 Question Paper (Stomatology)
Krok 1 - 2006 Question Paper (Stomatology)
 
Krok 2 - 2014 (Pediatrics)
Krok 2 - 2014 (Pediatrics)Krok 2 - 2014 (Pediatrics)
Krok 2 - 2014 (Pediatrics)
 

More from Bahaa A

2013krok
2013krok2013krok
2013krokBahaa A
 
pediatric сase history
pediatric сase historypediatric сase history
pediatric сase historyBahaa A
 
Sport medicine
Sport medicineSport medicine
Sport medicineBahaa A
 
Acute pancreatitis
Acute pancreatitisAcute pancreatitis
Acute pancreatitisBahaa A
 

More from Bahaa A (7)

2013krok
2013krok2013krok
2013krok
 
pediatric сase history
pediatric сase historypediatric сase history
pediatric сase history
 
Sport medicine
Sport medicineSport medicine
Sport medicine
 
Acute pancreatitis
Acute pancreatitisAcute pancreatitis
Acute pancreatitis
 
Urology
UrologyUrology
Urology
 
Diary
Diary Diary
Diary
 
2010
20102010
2010
 

Recently uploaded

Disha NEET Physics Guide for classes 11 and 12.pdf
Disha NEET Physics Guide for classes 11 and 12.pdfDisha NEET Physics Guide for classes 11 and 12.pdf
Disha NEET Physics Guide for classes 11 and 12.pdfchloefrazer622
 
Call Girls in Dwarka Mor Delhi Contact Us 9654467111
Call Girls in Dwarka Mor Delhi Contact Us 9654467111Call Girls in Dwarka Mor Delhi Contact Us 9654467111
Call Girls in Dwarka Mor Delhi Contact Us 9654467111Sapana Sha
 
General AI for Medical Educators April 2024
General AI for Medical Educators April 2024General AI for Medical Educators April 2024
General AI for Medical Educators April 2024Janet Corral
 
Software Engineering Methodologies (overview)
Software Engineering Methodologies (overview)Software Engineering Methodologies (overview)
Software Engineering Methodologies (overview)eniolaolutunde
 
Z Score,T Score, Percential Rank and Box Plot Graph
Z Score,T Score, Percential Rank and Box Plot GraphZ Score,T Score, Percential Rank and Box Plot Graph
Z Score,T Score, Percential Rank and Box Plot GraphThiyagu K
 
The basics of sentences session 2pptx copy.pptx
The basics of sentences session 2pptx copy.pptxThe basics of sentences session 2pptx copy.pptx
The basics of sentences session 2pptx copy.pptxheathfieldcps1
 
Web & Social Media Analytics Previous Year Question Paper.pdf
Web & Social Media Analytics Previous Year Question Paper.pdfWeb & Social Media Analytics Previous Year Question Paper.pdf
Web & Social Media Analytics Previous Year Question Paper.pdfJayanti Pande
 
The Most Excellent Way | 1 Corinthians 13
The Most Excellent Way | 1 Corinthians 13The Most Excellent Way | 1 Corinthians 13
The Most Excellent Way | 1 Corinthians 13Steve Thomason
 
Sanyam Choudhary Chemistry practical.pdf
Sanyam Choudhary Chemistry practical.pdfSanyam Choudhary Chemistry practical.pdf
Sanyam Choudhary Chemistry practical.pdfsanyamsingh5019
 
Holdier Curriculum Vitae (April 2024).pdf
Holdier Curriculum Vitae (April 2024).pdfHoldier Curriculum Vitae (April 2024).pdf
Holdier Curriculum Vitae (April 2024).pdfagholdier
 
Q4-W6-Restating Informational Text Grade 3
Q4-W6-Restating Informational Text Grade 3Q4-W6-Restating Informational Text Grade 3
Q4-W6-Restating Informational Text Grade 3JemimahLaneBuaron
 
Key note speaker Neum_Admir Softic_ENG.pdf
Key note speaker Neum_Admir Softic_ENG.pdfKey note speaker Neum_Admir Softic_ENG.pdf
Key note speaker Neum_Admir Softic_ENG.pdfAdmir Softic
 
fourth grading exam for kindergarten in writing
fourth grading exam for kindergarten in writingfourth grading exam for kindergarten in writing
fourth grading exam for kindergarten in writingTeacherCyreneCayanan
 
Sports & Fitness Value Added Course FY..
Sports & Fitness Value Added Course FY..Sports & Fitness Value Added Course FY..
Sports & Fitness Value Added Course FY..Disha Kariya
 
Class 11th Physics NEET formula sheet pdf
Class 11th Physics NEET formula sheet pdfClass 11th Physics NEET formula sheet pdf
Class 11th Physics NEET formula sheet pdfAyushMahapatra5
 
Arihant handbook biology for class 11 .pdf
Arihant handbook biology for class 11 .pdfArihant handbook biology for class 11 .pdf
Arihant handbook biology for class 11 .pdfchloefrazer622
 
APM Welcome, APM North West Network Conference, Synergies Across Sectors
APM Welcome, APM North West Network Conference, Synergies Across SectorsAPM Welcome, APM North West Network Conference, Synergies Across Sectors
APM Welcome, APM North West Network Conference, Synergies Across SectorsAssociation for Project Management
 
Russian Escort Service in Delhi 11k Hotel Foreigner Russian Call Girls in Delhi
Russian Escort Service in Delhi 11k Hotel Foreigner Russian Call Girls in DelhiRussian Escort Service in Delhi 11k Hotel Foreigner Russian Call Girls in Delhi
Russian Escort Service in Delhi 11k Hotel Foreigner Russian Call Girls in Delhikauryashika82
 
Student login on Anyboli platform.helpin
Student login on Anyboli platform.helpinStudent login on Anyboli platform.helpin
Student login on Anyboli platform.helpinRaunakKeshri1
 

Recently uploaded (20)

Disha NEET Physics Guide for classes 11 and 12.pdf
Disha NEET Physics Guide for classes 11 and 12.pdfDisha NEET Physics Guide for classes 11 and 12.pdf
Disha NEET Physics Guide for classes 11 and 12.pdf
 
Call Girls in Dwarka Mor Delhi Contact Us 9654467111
Call Girls in Dwarka Mor Delhi Contact Us 9654467111Call Girls in Dwarka Mor Delhi Contact Us 9654467111
Call Girls in Dwarka Mor Delhi Contact Us 9654467111
 
General AI for Medical Educators April 2024
General AI for Medical Educators April 2024General AI for Medical Educators April 2024
General AI for Medical Educators April 2024
 
Software Engineering Methodologies (overview)
Software Engineering Methodologies (overview)Software Engineering Methodologies (overview)
Software Engineering Methodologies (overview)
 
Z Score,T Score, Percential Rank and Box Plot Graph
Z Score,T Score, Percential Rank and Box Plot GraphZ Score,T Score, Percential Rank and Box Plot Graph
Z Score,T Score, Percential Rank and Box Plot Graph
 
The basics of sentences session 2pptx copy.pptx
The basics of sentences session 2pptx copy.pptxThe basics of sentences session 2pptx copy.pptx
The basics of sentences session 2pptx copy.pptx
 
Web & Social Media Analytics Previous Year Question Paper.pdf
Web & Social Media Analytics Previous Year Question Paper.pdfWeb & Social Media Analytics Previous Year Question Paper.pdf
Web & Social Media Analytics Previous Year Question Paper.pdf
 
The Most Excellent Way | 1 Corinthians 13
The Most Excellent Way | 1 Corinthians 13The Most Excellent Way | 1 Corinthians 13
The Most Excellent Way | 1 Corinthians 13
 
Sanyam Choudhary Chemistry practical.pdf
Sanyam Choudhary Chemistry practical.pdfSanyam Choudhary Chemistry practical.pdf
Sanyam Choudhary Chemistry practical.pdf
 
Holdier Curriculum Vitae (April 2024).pdf
Holdier Curriculum Vitae (April 2024).pdfHoldier Curriculum Vitae (April 2024).pdf
Holdier Curriculum Vitae (April 2024).pdf
 
Q4-W6-Restating Informational Text Grade 3
Q4-W6-Restating Informational Text Grade 3Q4-W6-Restating Informational Text Grade 3
Q4-W6-Restating Informational Text Grade 3
 
Key note speaker Neum_Admir Softic_ENG.pdf
Key note speaker Neum_Admir Softic_ENG.pdfKey note speaker Neum_Admir Softic_ENG.pdf
Key note speaker Neum_Admir Softic_ENG.pdf
 
fourth grading exam for kindergarten in writing
fourth grading exam for kindergarten in writingfourth grading exam for kindergarten in writing
fourth grading exam for kindergarten in writing
 
Sports & Fitness Value Added Course FY..
Sports & Fitness Value Added Course FY..Sports & Fitness Value Added Course FY..
Sports & Fitness Value Added Course FY..
 
Class 11th Physics NEET formula sheet pdf
Class 11th Physics NEET formula sheet pdfClass 11th Physics NEET formula sheet pdf
Class 11th Physics NEET formula sheet pdf
 
Arihant handbook biology for class 11 .pdf
Arihant handbook biology for class 11 .pdfArihant handbook biology for class 11 .pdf
Arihant handbook biology for class 11 .pdf
 
APM Welcome, APM North West Network Conference, Synergies Across Sectors
APM Welcome, APM North West Network Conference, Synergies Across SectorsAPM Welcome, APM North West Network Conference, Synergies Across Sectors
APM Welcome, APM North West Network Conference, Synergies Across Sectors
 
Russian Escort Service in Delhi 11k Hotel Foreigner Russian Call Girls in Delhi
Russian Escort Service in Delhi 11k Hotel Foreigner Russian Call Girls in DelhiRussian Escort Service in Delhi 11k Hotel Foreigner Russian Call Girls in Delhi
Russian Escort Service in Delhi 11k Hotel Foreigner Russian Call Girls in Delhi
 
Student login on Anyboli platform.helpin
Student login on Anyboli platform.helpinStudent login on Anyboli platform.helpin
Student login on Anyboli platform.helpin
 
Mattingly "AI & Prompt Design: Structured Data, Assistants, & RAG"
Mattingly "AI & Prompt Design: Structured Data, Assistants, & RAG"Mattingly "AI & Prompt Design: Structured Data, Assistants, & RAG"
Mattingly "AI & Prompt Design: Structured Data, Assistants, & RAG"
 

2008

  • 1. 1. A human body cools in water much C. Mature faster that in the air. What way of heat emission D. Atretic in water is much more efficient? E. Primary A. Convection 7. 2 hours after a skeletal extension was B. Heat radiation performed to a 27 year old patient with C. Heat conduction multiple traumas (closed injury of chest D. Sweat evaporation closed fracture of right thigh) his condition E. - abruptly became worse and the patient died 2. A driver who got a trauma in a road accident from acute cardiopulmonary decompensation. and is shocked has reduction of daily Histological examination of pulmonary urinary output down to 300 m1. What is the and cerebral vessels stained with Sudan III main pathogenetic factor of such diuresis revealed orange drops occluding the vessel change? lumen. What complication of polytrauma A. Drop of arterial pressure. was developed? B. Secondary hyperaldosteronism A. Air embolism C. Increased vascular permeability B. Thromboembolism D. Decreased number of functioning C. Fat embolism glomerules D. Microbal embolism E. Drop of oncotic blood pressure E. Gaseous embolism 3. Myocyte cytoplasm contains a big number 8. A patient complains of frequent diarrheas, of dissolved metabolites of glucose oxidation. especially after consumption of Name one of them that turns directly rich food, weight loss. Laboratory examination into a lactate: revealed steatorrhea; his feces were A. Pyruvate hypocholic. What might have caused such B. Glycerophosphate condition? C. Glucose 6-phosphate A. lnflammation of mucous membrane of D. Oxaloacetate small intestine E. Fructose 6-phosphate B. Obturation of biliary tracts 4. Continuous taking of some drugs foregoing C. Lack of pancreatic lipase the pregnancy increase the risk of giving D. Lack of pancreatic phospholipase birth to a child with genetic defects. What is E. Unbalanced diet this effect called? 9. A 12 year old child has intolerance to A. Mutagenic effect some foodstuffs. Their consumption causes B. Blastomogenic effect an allergic reaction in form of itching skin C. Fetotoxical effect eruptions. What antihistaminic drug should D. Embryotoxic effect be admistered so that the child could attend E. Teratogenic effect school? 5. A patient has yellow skin colour, dark urine A. Loratadine dark-yellow feces. What substance will B. Dimedrol have strengthened concentration in the C. Ephedrine blood scrum? D. Aminophylline A. Verdoglobin E. Diclofenac B. Conjugated bilirubin 10. Labelled amino acids alanine and C. Biliverdin tryptophane were injected to a mouse D. Unconjugated bilirubin in order to study localization of protein E. Mesobilirubin synthesis in its cells. The labelled amino acids 6. Examination of an ovary specimen stained will be accumulated near the following by hematoxylin-eosine revealed a follicle organellas: in which follicular epithelium consisted A. Smooth endoplasmic reticulum of 1-2 layers of cubic cells. There was also B. Ribosomes a bright red membrane around the ovocyte. C. Lysosomes What follicle is it? D. Cell centre A. Secondary E. Golgi apparatus B. Primordilal 11. As a result of prophylactic meedical
  • 2. examination a 7 year old boy was diagnosed B. Intensification of glucose absorption m with Lesch-Nyhan syndrome (only boys fall the bowels ill with it). The boy's parents are healthy but C. Gluconeogenesis activation his grandfather bv his mother's side suffers D. Glycogenogenesis activation from the same disease. What type or disease E. Inhibition of glycogen synthesis inheritance is it? 16. A man with a stab wound in the area of A. Dominant, sex-linked quadrilateral foramen applied to a doctor. B. Recessive, sex-linked Examination revealed that the patient was C. Autosomal recessive unable to draw his arm aside from his body. D. Autosomal dominant What nerve is most probably damaged? E. Semidominance A. N.axillaris 12. A patient ill with chronic cardiac B. N.radialis insufficiency was prescribed an C. N.subclavius average therapeutic dose of digoxin. Two D. Nmedianus weeks after begin of its taking there E. N ulnaris appeared symptoms of drug intoxication 17. Ultramicroscopical examination of (bradycardia, extrasystole, nausea). Name "dark" hepatocyte population in the cell the phenomenon that caused accumulation cytoplasm detected a developed granular of the drug in the organism? endoplasmic reticulum. What function has A. Material cumulation this organella in these cells? B. Tolerance A. Bile production C. Functional cumulation B. Deintoxicative function D. Idiosyncrasy C. Calcium ion depositing E. Tachyphylaxis D. Carbohydrate synthesis 13. Bacterioscopic examination of a smear E. Synthesis of blood plasma proteins from the pharynx of a diphtheria suspect 18. A 17 year old boy fell seriously ill, the revealed bacilli with volutine granules. What body temperature rose up to 38,5°C, there etiotropic drug should be chosen in this appeared cough, rhinitis, lacrimation, nasal case? discharges. What inflammation is it? A. Eubiotic A. Hemorrhagic B. Antidiphtheritic antitoxic serum B. Purulent C. Bacteriophage C. Fibrinous D. Diphtheritic anatoxin D. Catarrhal E. Interferon E. Serous 14. A culture of monkey cells (Vero) and 19. Researchers isolated 5 isoenzymic forms a group of mouse sucklings were infected of lactate dehydrogenase from the human with an inoculum taken from a child with blood serum and studied their properties. provisional diagnosis "enterovirus infection". What property indicates that the isoenzymic There was no cytopathic effect on the forms were isolated from the same enzyme? cell culture but mouse sucklings died. What A. The same electrophoretic mobility enteric viruses might have caused disease of B. Tissue localization this child? C. The same molecular weight A. Coxsackie B D. The same physicochemical properties B. Coxsackie A E. Catalyzation of the same reaction C. Unclassified enteric viruses 68-71 20. A 60 year old patient has impaired D. ECHO virus perception of high-frequency sounds. These E. Polioviruses changes were caused by damage of the 15. A patient ill with neurodermatitis has following auditory analyzer structures: been taking prednisolone for a long time. A. Tympanic membrane Examination revealed high rate of sugar in B. Main cochlea membrane near the oval his blood. This complication is caused by window the drug influence upon the following link C. Eustachian tube of carbohydrate metabolism: D. Middle ear muscles A. Activation of insulin decomposition E. Main cochlea membrane near the helicotrema
  • 3. 21. ECG of a patient with hyperfunction of D. Biotin thyroid gland showed heart hurry. It is indicated E. Vitamin A by depression of the following ECG 26. A patient ill with essential hypertension element: was recommended a drug that prevents A. QRS complex thrombosis. It is to be taken parenterally. B. R - R interval What drug is it? C. P – Q interval A. Protamine sulfate D. P – Q segment B. Amben E. P - T interval C. Heparin 22 A patient taking clonidine for essential D. Neodicumarin hypertension treatment was using alcohol E. Syncumar that caused intense inhibition of central 27. A 38 year old patient suffers from nervous system. What may it be connected rheumatism in its active phase. What with? laboratory characteristic of blood serum A. Intoxication is of diagnostic importance in case of this B. Idiosyncrasy pathology? C. Cumulation A. Uric acid D. Effect potentiating B. Creatinine E. Effect summation C. C-reactive protein 23. A patient is ill with diabetes mellitus D. Transferrin accompanied by hyperglycemia on E. Urea an empty stomach (7,2 millimole/l). The 28. A 28 year old man had a gunshot wound hyperglycemia rate can be retrospectively of shin that resulted in an ulcer from the side estimated (over the last 4-8 weeks before of the injury. What is the main factor of the examination) on the ground of the rate neurodystrophy pathogenesis in this case? of the following blood plasma protein: A. Microcirculation disturbance A.Albumin B. Infection B. Ceruloplasmin c. Tissue damage C. Fibrinogen D. Psychical stress D. Glycated hemoglobin E. Traumatization of peripheral nerve E. C-reactive protein 29. A 26 year old man is in the torpid shock 24. 6 months after labour a woman had Phase as a result of a car accident. In blood: uterine hemorrhage. Gynaecological examination 3,2· 109/1. What is the leading mechanism of of uterine cavity revealed a dark-red leukopenia development? tissue with multiple cavities resembling of A. Leikopoiesis inhibition a "sponge". Microscopic examination of a B. Redistribution of leukocytes m tumour revealed in blood lacunas atypic light bloodstream epithelial Langhans cells and giant cells C. Disturbed going out of mature leukocytes of syncytiotrophoblast. What tumour is it? from the marrow into the blood A. Adenocarcinoma D. Lysis of leukocytes in the blood-forming B. Cystic mole organs C. Fibromyoma E. Intensified elimination of leukocytes from D. Chorioepithelioma the organism E. Squamous cell nonkeratinous carcinoma 30. Parents of a 3 year old child have 25. A 3 year old child with symptoms of been giving him antibiotics with purpose of stomatitis, gingivitis and dermatitis of open preventing enteric infections for a long time. skin areas was delivered to a hospital. A month later the child's condition changed Examination revealed inherited disturbance for the worse. Blood examination revealed of neutral amino acid transporting in the apparent leukopenia and granulocytopenia. bowels. These symptoms were caused by the What is the most probable mechanism of deficiency of the following vitamin: blood changes? A. Niacin A. Autoimmune B. Pantothenic acid B. Myelotoxic C. Cobalamin C. Redistributive
  • 4. D. Age-specific traumatology department. Objectively: cut E. Hemolytic wound of palmar surface of left hand: middle 31. Examination of a 12 year old boy with phalanxes of II-V fingers don't bend. developmental lag revealed achondroplasia: What muscles are damaged? shortening of upper and lower limbs as a A. Palmar interosseous muscles result of growth disorder of epiphyseal cartilages B. Profound finger flexor of long tubal bones. This disease is: C. Dorsal interosseous muscles A. Acquired D. Lumbrical muscles B. Inherited, dominant E. Superficial finger flexor C. Inherited, sex-linked 36. A 65 year old man suffering from D. Congenital gout complains of kidney pain. Ultrasound E. Inherited, recessive examination revealed renal calculi. The 32. Hartnup disease is caused by point most probable cause of calculi formation mutation of only one gene which results in is the strengthened concentration of the disturbance of tryptophane absorption in following substance: the bowels and its resorption in the renal A. Uric acid tubules. It is the reason for disorder of B. Cholesterol both digestive and urination systems. What C. Bilirubin genetic phenomenon is observed in this D. Urea case? E. Cystine A. Polymery 37. A patient with frequent attacks of B. Complementary interaction stenocardia was prescribed sustak-forte to C. Pleiotropy be taken one tablet twice a day. At first D. Semidominance the effect was positive but on the second E. Codominance day stenocardia attacks resumed. What can 33. A 46 year old patient applied to a doctor explain inefficiency of the prescribed drug? complaining about joint pain that becomes A. Tachyphylaxis stronger the day before weather changes. B. Sensibilization Blood examination revealed strengthened C. Dependence concentration of uric acid. The most D. Cumulation probable cause of the disease is the intensified E. Idiosyncrasy disintegration of the following substance: 38. An aged man had raise of arterial A. Uridine triphosphate pressure under a stress. It was caused by B. Thymidine monophosphate activation of: C. Uridine monophosphate A. Functions of thyroid gland D. Adenosine monophosphate B. Sympathoadrenal system E. Cytidine monophosphate C. Functions of adrenal cortex 34. A 28 year old patient had high arterial D. Hypophysis function pressure, hematuria and facial edemata. In E. Parasympathetic nucleus of vagus spite of treatment renal insufficiency was 39. A cerebral trauma caused increased progressing. 6 months later the patient died ammonia generation. What amino acid from uremia. Microscopic examination participates in the excretion of ammonia of his kidneys and their glomerules revealed from the cerebral tissue? proliferation of capsule nephrothelium and A. Glutamic of podocytes with "demilune "formation, B. Lysine sclerosis and hyalinosis of glomerules. What C. Tyrosine disease corresponds with the described picture? D. Tryptophan A. Acute glomerulonephritis E. Valine B. Subacute glomerulonephritis 40. As a result of posttranslative modifications C. Nephrotic syndrome some proteins taking part in blood D. Chronic glomerulonephritis coagulation, particularly prothrombin, E. Acute pyelonephritis become capable of calcium binding. The 35. A 35 year old man with a trauma following vitamin takes part in this process: of his left hand was admitted to the A. B2
  • 5. B. B1 46. While palpating mammary gland of a C. A patient a doctor revealed an induration D. C in form of a node in the inferior medial E. K quadrant. Metastases may extend to the 41. A student takes notes of a lecture. Quality following lymph nodes: of his notes became significantly worse when his A. Superior diaphragmal neighbours began talking. What B. Profound lateral cervical type of conditional reflex inhibition was the C. Bronchopulmonary cause of it? D. Posterior mediastinal A. Protective E. Parasternal B. Differentiated 47. A gynaecologist was examining a patient C. External and revealed symptoms of genital tract D. Delayed inflammation. A smear from vagina contains E. Extinctive pyriform protozoa with a spine, flagella 42. A 7 vear old child is ill with bronchilis. It is at their front: there is also an undulating necessary to administer him an membrane. What disease can be suspected? antibacterial drug. What drug of fluoroquinolone A. Intestinal trichomoniasis group is CONTRA-INDICATED at B. Toxoplasmosis this age? C. Lambliasis A. Cyprofloxacin D. Urogenital trichomoniasis B. Sulfadimethoxine E. Balantidiasis C. Ampiox 48. In course of an experiment a big number D. Ampicillin of stem cells of red bone marrow was in E. Amoxicillin some way destructed. Regeneration of which 43. A patient is ill with hepatocirrhosis. cell populations in the loose connective State of antitoxic liver function can be tissue will be inhibited? characterized by examination of the following A. Of lipocytes substance exreted by urine: B. Of fibroblasts A. Uric acid C. Of pericytes B. Hippuric acid D. Of pigment cells C. Ammonium salts E. Of macrophags D. Amino acids 49. A pregnant woman had her blood group E. Creatinine identified. Reaction of erythrocyte agglutination 44. A patient suffering from chronic cardiac with standard serums of 0αβ (I), Bα insufficiency was recommended to undergo (III) groups didn't proceed with standard a prophylactic course of treatment with a serum of Aβ (II) group. The blood group cardiological drug from the group of cardiac under examination is: glycosides that is to be taken enterally. What A. Aβ (II) drug was recommended') B. AB (IV) A. Corglycon C. Bα (III) B. Cordiamin D. - C. Strophanthine E. 0αβ(I) D. Cordarone 50. Prophylactic medical examination of a E. Digoxin 36 year old driver revealed that his AP was 45. 48 hours after tuberculine test (Mantoux 150/90 mm Hg. At the end of working day he test) a child had a papule 10 mm in diameter usually hears ear noise, feels slight indisposition on the spot of tuberculine injection. What that passes after some rest. He was diagnosed hypersensitivity mechanism underlies these with essential hypertension. What changes? is the leading pathogenetic mechanism in A. Anaphylaxy this case? B. Granulomatosis A. Neurogenetic C. Antibody-dependent cytotoxicity B. Endocrinal D. Cellular cytotoxicity C. Reflexogenic E. Immunocomplex cytotoxicity D. Nephric
  • 6. E. Humoral "progressing muscular dystrophy" got his 51. A histological specimen of spleen urine tested. What compound will confirm shows a vessel with a wall consisting this diagnosis if found in urine? of endothelium and subendothelial layer, A. Kreatine median membrane is absent, exterior B. Calmodulin membrane inosculates with the layers of C. Collagen spleen connective tissue. What vessel is it? D. Myoglobin A. Artery of muscular type E. Porphyrin B. Capillary 57. While playing volleyball a sportsman C. Vein of muscular type made a jump and landed on the outside D. Arteriole edge of his foot. He felt acute pain in the E. Vein of non-muscular type talocrural joint, active movements are limited, 52. It was revealed that T-lymphocytes passive movements are unlimited but were affected by HIY. Virus enzyme reverse painful. A bit later there appeared a swelling transcriptase (RN A -dependent in the area of external ankle, the skin DNA polymerase) - catalyzes the synthesis became red and warm. What type of peripheral of: circulation disturbance is the case? A. Viral DNA on DNA matrix A. Arterial hyperemia B. Virus informational RNA on the matrix of B. Venous hyperemia DNA C. Stasis C. DNA on virus ribosomal RNA D. Embolism D. DNA on the matrix of virus mRNA E. Thrombosis E. mRNA on the matrix of virus protein 58. Autopsy of a man who had been working 53. Planned mass vaccination of all newborn as a miner for many years and died from 5-7 day old children against tuberculosis plays cardiopulmonary decompensation revealed an important role in tuberculosis prevention. that his lungs were airless, sclerosed, their In this case the following vaccine is apexex had emphysematous changes, the applied: lung surface was greyish-black, the incised A.BCG lung tissue was coal-black. What disease B. Adsorbed diphtheria vaccine caused death? C. Diphteria and tetanus toxoids and A. Silicosis pertussis vaccine B. Talcosis D. Diphtheria and tetanus anatoxin vaccine C. Anthracosis E. - D. Aluminosis 54. Life cycle of a cell includes the process E. Asbestosis of DNA autoreduplieation. As a result of it 59. Examination of a 55 year old woman monochromatid chromosomes turn into revealed under the skin of submandibular bichromatid area a movable slowly growing pasty ones. What period of cell cycle formation with distinct borders l,0x0,7 cm does this phenomenon fall into? large. Histological examination revealed A. G2 Iipocytes B. Go that form segments of diffrent forms C. G1 and sizes separated from each other by D. S thin layers of connective tissue with vessels. E. M What is the most probable diagnosis? 55. Rest potential of a cell equals -80 mV. A. Lipoma At what stage of action potential did the B. Fibroma membrane potential equal +30 mV? C. Liposarcoma A. After depolarization D. Fibrosarcoma B. After hyperpolarization E. Angioma C. Reverse polarization 60. A patient is 44 years old. Laboratory D. - examination of hrs blood revealed that E. Depolarization content of proteins in plasma was 40 g/l. 56. A patient with suspected diagnosis What influence will be exerted on the
  • 7. transcapillary water exchange? C. M.rhomboideus minor A. Exchange will stay unchanged . D. M.latissimus dorsi B. Filtration will be increased, reabsorption- E. M.sternocleidomastoideus decreased 65. A patient with a knife wound in the C. Filtration will be decreased, reabsorption left lumbal part was delivered to the - increased emergency hospital. In course of operation D. Both filtration and reabsorption will be a surgeon found that internal organs were increased not damaged but the knife injured one of E. Both filtration and reabsorption will be muscles of renal pelvis. What muscle is it? decreased A. Iliac muscle 61. A 30 year old woman has face edemata. B. Abdominal internal oblique muscle Examination revealed proteinuria (5,87 g/l), C. Erector muscle of spine hypoproteinemia, dysproteinemia, D. Greater psoas muscle hyperlipidemia. E. Abdominal external oblique muscle What condition is the set of these 66. A 56 year old patient came to a hospital symptoms typical for? with complaints about general weakness, A. Chronic renal failure tongue pain and burning, sensation of limb B. Nephritic syndrome numbness. In the past he underwent resection of C. Nephrotic syndrome forestomach. In blood: Hb- 80 g/l; D. Chronic pyelonephritis erythrocytes – 2,0*1012/l; colour index - 1.2, E. Acute renal failure leukocytes – 3,5*109/l. What anemia type is 62. Vitamin B1 deficiency results in disturbance it? of oxidative decarboxylation of α-ketoglutaric A. Hemolytic acid. This will disturb synthesis B. B12-folate deficient of the following cocnzyme: C. Aplastic A. Nicotinamide adenine dinucleotide D. Posthemorrhagic (NAD) E. Iron-deficient B. Lipoic acid 67. A patient ill with thrombophlehitis of his C. Flavine adenine dinucleotide (FAD) lower limbs had chest pain, blood spitting, D. Thiamine pyrophosphate progressing respiratory insufficiency that led E. Coenzyme A to his death. Autopsy diagnosed multiple 63. Skin of a man who died from cardiac lung a infarctions. What is the most probable insufficiency has an eruption in form cause of their development? of spots and specks. There are also A. Thromboembolism of pulmonary artery bedsores in the area of sacrum and spinous branches vertebral processes. Microscopical B. Thrombosis of bronchial arteries examination of CNS, skin, adrenal glands C. Thrombosis of pulmonary artery branches revealed in the vessels of microcirculatory D. Thrombosis of pulmonary veins bed and in small arteries destructiveproliferative E. Thromboembolism of bronchial arteries endothrombovasculitis with 68. A concentrated solution of sodium Popov's granulomas; interstitial myocarditis. chloride was intravenously injected to an What diagnosis corresponds with the described animal. This caused decreased reabsorption picture? of sodium ions in the renal tubules. It is the A. Spotted fever result of the following changes of hormonal B. Q fever secretion: C. Nodular periarteritis A. Aldosterone increase D. Enteric fever B. Reduction of atrial natriuretic factor E. HIV C. Vasopressin increase 64. A man with an injury of the dorsal area D. Aldosterone reduction of his neck was admitted to the resuscitation E. Vasopressin reduction department. What muscle occupies this 69. A patient had hemorrhagic stroke. Blood area? examination revealed strengthened kinin A. M.trapezius concentration.The patient was prescribed B. M.scalenus anterior contrical. It was administered in order to
  • 8. inhibit the following proteinase: heart structure: A. Trypsin A. Ventricle B. Chemotrypsin B. Sinoatrial node C. Pepsin C. Atrioventricular node D. Collagenase D. His' bundle E. Kallikrein E. Atrium 70. Packed cell volume of a man was 40% 76. Examination of coronary arteries before the trauma. What packed cell volume revealed atherosclerotic calcified plaques will be observed 24 hours after blood loss of closing vessel lumen by 1/3. The muscle has 750 ml? multiple whitish layers of connective tissue. A.40% What process was revealed in the myocardium? B.30% A. Postinfarction cardiosclerosis C.55% B. Myocardium infarction D.50% C. Diffusive cardiosclerosis E.45% D. Myocarditis 71. On the 2-3rd day after stomach resection E. Tiger heart intestinal peristalsis wasn't restored. What 77. A patient with cholelithiasis fell ill with is to be administered for stimulation of mechanic jaundice. Examination revealed gastrointestinal tract? that the stone was in the common bile A. Atropine sulfate duct. What bile-excreting ducts make up the B. Prasosin obturated duct? C. Acetylcholine A. Ductus hepaticus sinister et ductus cysticus D. Proserin B. Ductus hepaticus dexter et sinister E. Cyclodole C. Ductus hepaticus communis et ductus 72. An ophthalmologist used a 1% mesaton choledochus solution for the diagnostic purpose (pupil dilation D. Ductus hepnriclls C0l11111W1lS et ductus for eve-ground examination). What is cllOledochlls the cause of mydriasis induced by the drug? E. Ductus hepaticus dexter ct ductus cysticus A. Activation of α1 adrenoreceptors 78. 2 years ago a patient underwent resection B. Activation of β1 adrenoreceptors of pyloric part of stomach. He complains C. Activation of M-cholinoreceptors of weakness, periodical dark shadows D. Block of α1 adrenoreceptors beneath his eyes, dyspnea. In blood: Hb - 70 E. Activation of α2 adrenoreceptors g/l erythrocytes - 3,0 * 1012 /1, colour index 73. A patient with disturbed cerebral circulation - 0,7. What changes of erythrocytes in blood has problems with deglutition. smears are the most typicat for this condition? What part of brain was damaged? A. Macrocytes A. Interbrain B. Ovalocytes B. Cervical part of spinal cord C. Megalocytes C. Midbrain D. Schizocytes D. Brainstem E. Microcytes E. Forebrain 79. A 6 year old child was delivered to a 74. A 16 year old boy after an illness has hospital. Examination revealed that the child diminished function of protein synthesis in couldn't fix his eves, didn't keep his eyes liver as a result of vitamin K deficiency. It on toys, eye ground had the cherry-red spot will cause disturbance of: sign. Laboratory analyses showed that brain, A. Blood coagulation Iiver and spleen had high rate of ganglioside B. Anticoagulant generation glycometide. What congenital disease is the C. Erythropoietin secretion child ill with? D. Erythrocyte sedimentation rate A. Wilson's syndrome E. Osmotic blood pressure B. MacArdle disease 75. An isolated cell of human heart C. Niemann-Pick disease automatically generates excitement D. Tay-Sachs disease impulses with frequency of 60 times per minute. E. Turner's syndrome This cell was taken from the following 80. In course of an experiment a peripheral
  • 9. section of vagus of an expiremental animal B. Retinol acetate is being stimulated. What changes will be C. Levamisole observed? D. Prednisolone A. Heart rate fall E. Retabolil B. Heart hurry 86. A stomatologist injected a patient with C. Increase of respiration rate a certain drug in order to reduce salivation D. Bronchi dilation during tooth filling. What drug is it? E. Pupil dilation A. Mesaton 81. A patient with bacterial pneumonia B. Proserin was prescribed benzylpenicillin. What is the C. Atropine sulfate mechanism of its antibacterial effect? D. Adrenaline hydrochloride A. Abnormal permeability of cytoplasmic E. Pilocarpine hydrochloride membrane 87. Vagus branches that innervate heart are B. Inhibition of SH-groups of microorganism being stimulated in course of an experiment. enzymes As a result of it the excitement conduction C. Inhibition of synthesis of microorganism from atria to the ventricles was brought to a wall stop. It is caused by electrophysical changes D. Inhibition of intracellular protein in the following structures: synthesis A. His' bundle E. Antagonism with p-amino-benzoic acid B. Atria 82.Blood minute volume of a 30 year old woman C. Atrioventricular node at rest is 5 l/m. What blood volume D. Sinoatrial node is pumped through the pulmonary vessels E. Ventricles per minute? 88. A doctor examined a child and revealed A. 3,75 l symptoms of rachitis. Development of this B. 1,5 l desease was caused by deficiency of the C. 2,5 l following compound: D. 2,0 l A. Naphtaquinone E. 5,1 B. Retinol 83. In course of an experiment a toad's right C. Biotin labyrinth was destroyed. It will cause D. 1,25 [OH]-dichydroxycholecalciferol amyotonia of the following muscles: E. Tocopherol A. Right and left extensors 89. Histological examination of a skin tissue B. Right extensors sampling revealed granulomas consisting C. Left flexors of macrophagal nodules with lymphocytes D. Right flexors and plasmatic cells. There are also some big E. Left extensors macrophages with fatty vacuoles containing 84. A patient died from cardiopulmonary causative agents of a disease packed up in decompensation. Histological examination form of spheres (Virchow's cells). Granulation revealed diffused pulmonary lesion together tissue is well vascularized. What disease with interstitial edema. infiltration of tissue is this granuloma typical for? by limphocytes, macrophages, plasmocytes, A. Rhinoscleroma pulmonary fibrosis, panacinar emphysema. B. Syphilis What disease corresponds with the described C. Lepra picture? D. Tuberculosis A. Pulmonary atelectasis E. Glanders B. Bronchial asthma 90. A patient consulted a stomatologist C. Fibrosing alveolitis about purulent inflammation of his gums. D. Chronic bronchitis What drug will be the most effective if it E. Bronchopneumonia is suspected that a causative agent is an 85. In order to accelerate healing of a radiation anaerobe? ulcer a vitamin drug was administered. A. Oxacillin sodium What drug is it? B. Gentamicin A. Methyluracil C. Nitroxoline
  • 10. D. Co-trimoxazole diagnosis? E. Metronidazole A. Pediculosis 91. Microscopy of stained (Ziehl-Neelsen B. Myiasis staining) smears taken from the sputum C. Scabies of a patient with chronic pulmonary disease D. Demodicosis revealed red bacilli. What property E. Phthiriasis of tuberculous bacillus was shown up? 96. Examination of a patient with pustular A. Acid resistance skin lesions allowed to isolate a causative B. Alcohol resistance agent that forms in the blood agar roundish C. Capsule formation yellow middle-sized colonies surrounded by D. Sporification haemolysis zone. Smears from the colonies E. Alkali resistance contain irregular-shaped clusters of grampositive 92. The upper lobe of the right lung cocci. The culture is oxidase- and is enlarged, grey and airless, the inscision catalase-positive, ferments mannitol and surface is dripping with turbid liquid, synthesizes plasmocoagulase. What causative the pleura has many fibrinogenous films; agent was isolated? microscopical examination of alveoles A. Staphylococcus saprophyticus revealed exudate containing neutrophils, B. Staphylococcus aureus desquamated alveolocytes and fibrin fibers. C. Staphylococcus epidermidis The bronchus wall is intact. What is the most D. Streptococcus agalactiae probable diagnosis? E. Streptococcus pyogenes A. Focal pneumonia 97. Inoculum from pharynx of a patient B. Pulmonary abscess ill with angina was inoculated into bloodtellurite C. Inf1uenzalpneumonia agar. It resulted in growth of grey, radially D. Croupous pneumonia striated (in form of rosettes) colonies 4-5 -mm in E. Interstitial pneumonia dian1eter. Gram-positive bacilli with clublike 93. A patient has a decreased vasopressin thickenings on their ends synthesis that causes polyuria and as a result placed in form of spread wide apart fingers of it evident organism dehvdratation. What are visible by microscope. What microorganisms is the mechanism of polyuria development? are these? A. Reduced tubular reabsorption of protein A. Streptococci B. Reduced tubular reabsorption of Na ions B. Botulism clostridia C. Reduced glucose reabsorption C. Diphtheria corynebacteria D. Reduced tubular reabsorption of water D. Streptobacilli E. Acceleration of glomerular filtration E. Diphtheroids 94. A patient of surgical department 98. A man with a wound of his limb that had complains about pain in the small of her been suppurating for a long time died from back and in the lower part of her belly; painful intioxication. Autopsy revealed extreme and frequent urination. Bacteriological emaciation, dehydration, brown atrophy of examination of urine revealed gramnegative liver. myocardium, spleen and cross-striated oxidase-positive rod-like bacteria muscles as well as renal amyloidosis. What forming greenish mucoid colonies with. specific diagnosis corresponds with the described picture? smell. What causative agent can it be? A. Chroniosepsis A. Mycoplasma pneumonie B. Brucellosis B. E.coli C. Chernogubov's syndrome C. Pseudomonas aeruginosa D. Septicopyemia D. Proteus mirabilis E. Septicemia E. Str.pyogenes 99. In course of severe respiratory viral 95. A patient has acne on his face. Microspcopic infection there appeared clinical signs of examination of scrapings from progressing cardiac insufficiency that caused the affected areas revealed living porrect death of a patient in the 2nd week of disease. vermiform arthropoda 0,2-0,5 mm large with Autopsy revealed that the heart was four pairs of short extremities in the front sluggish, with significant cavity dilatation. part of their bodies. What is the laboratory Histological examination of myocardium
  • 11. revealed plephora of microvessels and diffuse D. Sodium infiltration of stroma by lymphocytes E. Calcium and histiocytes. What disease corresponds 104. An unconscious young man with signs with the described picture? of morphine poisoning entered admission A. Stenocardia office. His respiration is shallow and B. Cardiomyopathy infrequent which is caused by inhibition of C. Acute coronary insufficiency respiratory centre. What type of respiratory D. Myocardium infarction failure is it? E. Myocarditis A. Ventilative restrictive 100. A patient consulted a doctor about B. Perfusive symmetric dermatitis of open skin areas. It C. Ventilative obstructive was found out that the patient lived mostly D. Ventilative dysregulatory on cereals and ate too little meat, milk and E. Diffusive eggs. What vitamin deficiency is the most 105. A patient being treated for viral evident? hepatitis type B got symptoms of hepatic A. Nicotinamide insufficiency. What blood changes indicative B. Biotin of protein metabolism disorder will be C. Tocopherol observed in this case? D. Folic acid A. Absolute hyperfibrinogenemia E. Calciferol B. Absolute hyperalbuminemia 101. A patient with clinical signs of C. Absolute hypoalbuminemia encephalitis was delivered to the infectious D. Proteinic blood composition is unchanged diseases hospital. Anamnesis registers a tick E. Absolute hyperglobulinemia bite. Hemagglutination-inhibition reaction 106. A patient ill with essential arterial helped to reveal antibodies to the causative hypertension had a hypertensic crisis that agent of tick-borne encephalitis in the dilution resulted in an attack of cardiac asthma. 1:20 which is not diagnostic. What actions What is the leading mechanism of cardiac should the doctor take after he had got such ins insufficiency in this case? result? A. Absolute coronary insufficiency A. To repeat examination with another B. Heart overload caused by increased blood diagnosticum volume B. To repeat the examination with scrum C. Heart overload caused by high pressure taken 10 days later D. Myocardium damage C. To deny diagnosis of tick-borne encephalitis E. Blood supply disturbance D. To examine the same serum 107. Surgical removal of a part of stomach E. To apply more sensitive reaction resulted in disturbed absorption of vitamin 102. Parents of a 10 year old boy consulted B12 , it is excreted with feces. The patient a doctor about extension of hair-covering, was diagnosed with anemia. What factor is growth of beard and moustache, low voice. necessary for absorption of this vitamin? Intensified secretion of which hormone must A. Gastromucoprotein be assumed? B. Hydrochloric acid A. Of testosterone C. Gastrin B. Of somatotropin D. Folic acid C. Of progesterone E. Pepsin D. Of oestrogen 108. A 45 year old man consulted a doctor E. Of cortisol about a plaque-like formation on his neck. 103. An animal with aortic valve insufficiency Histological examination of a skin bioptate got hypertrophy of its left heart ventricle. revealed clusters of round and oval tumour Some of its parts have local contractures. cells with a narrow border of basophilic What substance accumulated in the cytoplasm resembling of cells of basal epidermal myocardiocytes caused these contractures? layer. What tumour is it? A. Lactic acid A. Epidermal cancer B. Potassium B. Hydroadenoma C. Carbon dioxide C. Trichoepithelioma
  • 12. D. Basal cell carcinoma 114. The territory of an old burial ground for E. Syringoadenoma animal refuse that hasn't been used for over 109. A 45 year old woman is ill with breast 50 years is meant for house building. But soil cancer. Her left arm has symptoms of investigation showed the presense of viable lymphatic system insufficiency - limb edema, spores of a causative agent causing a very lymph node enlargement. What form of dangerous disease. What microorganism might lymphatic circulation insufficiency is it? have been preserved in soil for such a A. Mechanic insufficiency long period of time? B. - A. Bacillus anthracis C. Dynamic insufficiency B. Francisella tularensis D. Combined insufficiency C. Mycobacterium bovis E. Resorption insufficiency D. Brucella abortus 110. Mother of a 2 year old child consulted E. Yersinia pestis a stomatologist. In the period of pregnancy 115. During starvation muscle proteins she was irregularly taking antibiotics for an break up into free amino acids. These infectious disease. Examination of the child compounds will be the most probably revealed incisor destruction, yellow enamel, involved into the following process: brown rim around the dental cervix. What A. Glycogenolysis drug has apparent teratogenic effect? B. Decarboxylation A. Ampiox C. Gluconeogenesis in muscles B. Octadine D. Gluconeogenesis in liver C. Furosemide E. Synthesis of higher fatty acids D. Doxacycline 116. A 62 year woman complains of frequent E. Xantinol nicotinate pain attacks in the area of her chest 111. A man who has been taking a drug for and backbone, rib fractures. Her doctor a long time cannot withhold it because this suspected myeloma (plasmocytoma). What causes impairment of psychic, somatic and of the following laboratory characteristics vegetative functions. Name the syndrome will be of the greatest diagnostic of different disturbances caused by drug importance? discontinuation: A. Idiosyncrasy A. Hypoproteinemia B. Sensibilization B. Hyperalbuminemia C. Tachyphylaxis C. Hypoglobulinemia D. Abstinence D. Paraproteinemia E. Cumulation E. Proteinuria 112. A chemical burn caused esophagus 117. The cerebrospinal fluid is being examined stenosis. Difficulty of ingestion led to the for the purpose of diffrential meningitis abrupt loss of weight. In blood: 3,0 * 1012/l, diagnostics. At what site is the lumbal Hb - 106 g/,. crude protein - 57 g/l. What puncture safe? type of starvation is it? A. L II - L III A. Water B. L V – S I B. Incomplete C. Th XIl - L I C. Complete D. L III – L IV D. Proteinic E. L I - L II E. Absolute 118. Power inputs of a man were measured. 113. As a result of long-term starvation In what state was this man if his power the glomerular filtration of a man was inputs were lower than basal metabolism? accelerated by 20%. The most probable A. Nervous tension cause of filtration changes under such conditions B. Sleep is: C. Rest A. Fall of oncotic pressure of blood plasma D. Relaxation B. Increase of renal plasma flow E. Simple work C. Growth of filtration coefficient 119. Vagi of an experimental animal were D. Rise of systemic arterial pressure cut on both sides. What respiration changes E. Increased permeability of renal filter will be observed?
  • 13. A. It will become shallow and frequent C. Heterotransplantation B. It will become shallow and infrequent D. Isotransplantation C. No changes will be observed E. Allotransplantation D. It will become deep and infrequent 125. A 40 year old man noticed a reddening E. It will become deep and frequent and an edema of skin in the area of his neck 120. Pharmacological effects of antidepressants that later developed into a small abscess. arc connected with inhibition The incised focus is dense, yellowish-green. of an enzyme catalyzing biogenic amines The pus contains white granules. Histological noradrenaline and serotonine in the examination revealed drusen of a fungus, mitochondrions plasmatic and xanthome cells, macrophages. of cerebral neurons. What What type of mycosis is the most probable? enzyme participates in this process? A. Sporotrichosis A. Lyase B. Actinomycosis B. Monoamine oxidase C. Aspergillosis C. Transaminase D. Candidosis D. Peptidase E. Coccidioidomycosis E. Decarboxylase 126. An embryo displays disturbed process 121. A pathological process in bronchi of dorsal mesoderm segmentation and somite resulted in epithelium desquamation. What formation. What part of skin will have cells will regenerate bronchial epithelium? developmental abnormalities? A. Intercalary A. Epidermis B. Ciliate B. Derma C. Goblet C. Sudoriferous glands D. Basal D. Sebaceous glands E. Endocrinal E. Hair 122. Examination of a man who had been 127. In course of a conditional experiment working hard under higher temperature the development of mesenchyma cells was of the environment revealed abnormal completely inhibited. Development of the quantity of blood plasma proteins. What following muscular tissue will be disturbed: phenomenon is the case? A. Cardiac muscular tissue A. Absolute hyperproteinemia B. Smooth muscular tissue B. Paraproteinemia C. Skeletal muscular tissue C. Relative hyperproteinemia D. Neural muscular tissue D. Absolute hypoproteinemia E. Epidermal muscular tissue E. Dysproteinemia 128. A girl is diagnosed with adrenogenital 123. A 70 year old man IS ill with syndrome (pseudohermaphroditism). This vascular atherosclerosis of lower extremities pathology was caused by hypersecretion of and coronary heart disease. Exan1ination the following adrenal hormone: revealed disturbance of lipidic A. Adrenalin blood composition. The main factor of B. Estrogen atherosclerosis pathogenesis is the excess C. Aldosterone of the following lipoproteins: D. Androgen A. Low-density lipoproteins E. Cortisol B. Cholesterol 129. Golgi complex exports substances from C. Chylomicrons a cell due to the fusion of the membrane D. High-density lipoproteins saccule with the cell membrane. The saccule E. Intermediate density lipoproteins contents flows out. What process is it? 124. For the purpose of myocardium A. Exocytosis infarction treatment a patient was injected B. Facilitated diffusion with embryonal stem cells derived from this C. Endocytosis very patient by means of therapeutic cloning. D. Active- transport What transplantation type is it? E. All answers arc false A. Autotransplantation 130. An oncological patient was prescribed B. Xenotransplantation methotrexate. With the lapse of time target
  • 14. cells of the tumour lost susceptibility to this face, hirsutism, AP is 165/100 mm Hg, height drug. There is change of gene expression of - 164 cm, weight - 103 kg; the fat is the folowing enzyme: mostly accumulated on her neck, thoracic A. Thiaminase girdle, belly. What is the main pathogenetic B. Deaminase mechanism of obesity? C. Dehydrofolate reductase A. Increased production of glucocorticoids D. Folate decarboxylase B. Reduced glucagon production E. Folate oxidase C. Increased insulin production 131. An infant has apparent diarrhea resulting D. Reduced production of thyroid hormones from improper feeding. One of the main E. Increased mineralocorticoid production diarrhea effects is plentiful excretion of sodium 136. A 20 year old patient died from bicarbonate. What form of acid-base intoxication 8 days after artificial illegal balance disorder is the case? abortion performed in her 14-15th week of A. Metabolic alkalosis pregnancy. Autopsy of the corpse revealed B. Respiratory alkalosis yellowish colour of eye sclera and of skin, C. No disorders of acid-base balance will be necrotic suppurative endometritis, multiple observed pulmonary abscesses, spleen hyperplasia D. Respiratory acidosis with a big number of neutrophils in its sinuses. E. Metabolic acidosis What complication after abortion was 112. Workers of a hothouse farm work under developed? conditions of unfavourable microclimate: air A. Chroniosepsis temperature is +37°C relative humidity is B. Hemorrhagic shock 90%, air speed is 0,2 m/s. The way of heat C. Septicopyemia emission under these conditions will be: D. Septicemia A. Radiation E. Viral hepatitis type A B. Convection 137. There are several groups of molecular C. Heat conduction mechanisms playing important part in D. All the ways pathogenesis of insult to cells which contributes E. Evaporation to the pathology development. What 133. Examination of a 70 year old patient processes are stimulated by proteinic rrevealed insulin-dependent diabetes. What drug damage mechanisms? should be administered? A. Phospholipase activation A. Glibenclamid B. Osmotic membrane distension B. Parathyroidin C. Enzyme inhibition C. Insulin D. Lipid peroxidation D. Mercazolilum E. Acidosis E. Cortisone 138. A patient takes digoxin for treatment 134. A 59 year old patient is a plant manager. of cardiac insufficiency. What diuretic may After the tax inspection of his plant he felt increase digoxin toxicity due to the intensified intense pain behind his breastbone irradiating excretion of K+ ions? to his left arm. 15 minutes later his condition A. Lisinopril came to normal. Which of the possible B. Panangine mechanisms of stenocardia development is C. Siliborum the leading in this case? D. Spironolactone A. High catecholamine concentration m E. Hydrochlorothiazide blood 139. A patient ill with amebiasis was prescribed B. Coronary atherosclerosis a certain drug. The use of alcohol C. Functional heart overload together with this drug is contra-indicated D. Coronary thrombosis because the drug inhibits metabolism of E. Intravascular aggregation of blood ethyl alcohol. What drug is it? corpuscles A. Clonidine 135. A 44 year old woman complains of B. Metronidazole general weakness, heart pain, significant C. Reserpine increase of body weight. Objectively: moon D. Diazepam
  • 15. E. Aminazine dysentery and was once more infected with 140. While performing an operation in the the same causative agent. What is such area of axillary crease a surgeon has to define infection form called? an arterial vessel surrounded by fascicles A. Persisting infection of brachial plexus. What artery is it? B. Chronic infection A. A.axillaris C. Recidivation B. A.profunda brachii D. Superinfection C. A. vertebralis E. Reinfection D. A.subscapularis 146. A 30 year old woman has applied a lipstick E. A. transversa colli with a fluorescent substance for a 141. A patient with nephrotic syndrome has long time. Then she got a limited erythema massive edemata of his face and limbs. What and slight peeling on her lip border, later is the leading pathogenelic mechanism of there appeared transversal striae and cracks. edemata development? Special methods of microscopic examination A. Rise of hydrodynamic blood pressure of the affected area helped to reveal B. Lymphostasis sensibilized lymphocytes and macrophages C. Increase of vascular permeability in the connective tissue: cytolysis. What D. Increase of lymph outflow type of immunological hypersensitivity was E. Drop of oncotic blood pressure developed? 142. A patient underwent an operation on A. III type (immune complex cytotoxicity) account of gall bladder excision that resulted B. I type (reaginic) in obstruction of Ca absorption through the C. IVtype (cellular cytotoxicity) bowels wall. What vitamin will stimulate this D. II type (antihody cytotoxicity) process? E. Granulomatosis A.C 147. Autopsy of a man who died from B. B12 the sepsis in his femoral hone revealed C. D3 phlegmonous inflammation that affected the D. K marrow, haversian canals and periosteum. E. PP Under the periosteum there are multiple 143. While studying maximally spiralized abscesses, adjoining soft tissues of thigh also chromosomes of human karyotype the have signs of phlegmonous inflammation. process of cell division was stopped in the What pathological process was described? following phase: A. Chronic henlatogenous osteomielitis A. Telophase B. Osteoporosis B. Interphase c. - C. Anaphase D. Acute hemalogenous osteomyelitis D. Metaphase E. Osteopetrosis E. Prophase 148. A patient was diagnosed with autoimmune 144. A patient ill with tuberculosis died hemolitic cytotoxic anemia. What from progressing cardiopulmonary substances are antigens in II type allergic decompensation. Autopsy in the area of reactions? the right lung apex revealed a cavity 5 A. Antibiotics cm in diameter communicating with lumen B. Hormones of a segmental bronchus. On the inside C. Serum proteins cavity walls are covered with caseous D. Modified receptors of cell membranes masses with epithelioid and Langhans cells E. Inflammation modulators beneath them. What morphological form of 149. Rats being under stress have muscular tuberculosis is it? hypertonia and high arterial pressure, high A. Acute focal tuberculosis glucose concentration in blood and intensified B. Infiltrative tuberculosis secretion of corticotropin and corticosteroids. C. Caseous pneumonia In what stress phase are these animals? D. Acute cavernous tuberculosis A. Exhaustion E. Tuberculoma B. Terminal 145. A patient recovered from Sonne C. Antishock phase
  • 16. D. Erectile 155. A 38 year old patient with full-blown E. Shock phase jaundice, small cutaneous hemorrhages, 150. It was proved that a molecule of general weakness and loss of appetite immature mRNA (precursor mRNA) underwent puncture biopsy of liver. Histological contained more triplets than amino acids examination revealed disseminated found in the synthesized protein. The dystrophy, hepatocyte necrosis, reason for that is that translation is normally Councilman's bodies. Lobule periphery preceded by: has signs of significant infiltration by A. Processing lymphocytes, there are also individual B. Initiation multinuclear C. Reparation hepatocytes. What is the most D. Replication probable diagnosis? E. Mutation A. Acute alcoholic hepatitis 151. A patient who has been treated in a B. Toxic degeneration of liver neural clinic and has been taking a sedative C. Acute viral hepatitis for a long time got the following complication: D. Chronic hepatitis cough, rhinitis, epiphora. What drug E. Miliary hepatic cirrhosis caused these disturbances? 156. A patient with acute morphine poisoning A. Sodium bromide was delivered to a hospital. What specific B. Valerian narcotic antagonist should be chosen in this C. Phenazepam case? D. Reserpine A. Unithiol E. Diazepam B. Paracetamol 152. Examination of a newborn boy's C. Digoxin genitalia revealed an urethral hiatus that D. Methacin opens on the undersite of his penis. What E. Naloxone malformation is it? 157. Autopsy of a man ill with severe A. Cryptorhidism hypothyroidism revealed that connective tissue, B. Epispadia organ stroma, adipose and cartilaginous C. Hermaphroditism tissues were swollen, semitransparent, D. Monorchism mucus-like. Microscopic examination of tissues E. Hypospadias revealed stellate cells having processes 153. A patient that entered the admission office with mucus between them. What type of had the following signs of acute cardiac dystrophy is it? insuffiency: paleness, acrocyanosis, frequent A. Stromal-vascular carbohydrate shallow respiration. What drug is B. Strornal-vascular proteinaceous indicated in this case? C. Parenchymatous adipose A. Corglycon D. Parenchymatous proteinaceous B. Adrenaline hydrochloride E. Stromal-vascular adipose C. Nitroglycerine 158. In order to estimate toxigenity of diphtheria D. Cordiamin agents obtained from patients the E. Digitoxin cultures were inoculated on Petri dish with 154. In order to speed up healing of a wound nutrient agar on either side of a filter paper of oral mucosa a patient was prescribed a strip that was put into the centre and moistened drug that is a thermostable protein occuring with antidiphtheric antitoxic serum. in tears, saliva, mother's milk as well as in After incubation of inoculations in agar the a new-laid hen's egg. It is known that this strip-like areas of medium turbidity were protein is a factor of natural resistance of an found between separate cultures and the organism. What is it called? strip of filter paper. What immunological A. Complement reaction was conducted? B. Imanine A. Rings precipitation reaction C. lnterleukin B. Agglutination reaction D. Interferon C. Precipitation gel reaction E. Lysozyme D. Coomb's test
  • 17. E. Opsonization reaction caused complete disruption of spinal cord at 159. A patient ill with enteritis accompanied the level of the first cervical segment. What by massive diarrhea has low water rate in respiration changes will be observed? the extracellular space, high water rate inside A. No changes will be observed the cells and low blood osmolarity. What B. It will become infrequent and deep is such disturbance of water-electrolytic C. Thoracic respiration will be maintained, metabolism called? diaphragmal respiration will disappear A. Hyperosmolar hypohydration D. It will come to a standstill B. Hyperosmolar hyperhydration E. Diaphragmal respiration will be maintained, C. Osmolar hypohydration thoracic respiration will disappear D. Hypo-osmolar hyperhydration 165. Systemic arterial pressure of an adult E. Hypo-osmolar hypohydration dropped from 120/70 to 90/50 mm Hg 160. In course of a preventive examination that led to reflectory vasoconstriction. The of a miner a doctor revealed changes of vasoconstriction will be maximal in the cardiovascular fitness which was indicative following organ: of cardiac insufficiency at the compensation A. Bowels stage. What is the main proof of cardiac B. Heart compensation? C. Kidneys A. Myocardium hypertrophy D. Adrenals B. Tachycardia E. Brain C. Cyanosis 166. A pregnant woman was registered in D. Dyspnea an antenatal clinic and underwent complex E. Rise of arterial pressure examination for a number of infections. 161. It was found out that some compounds, Blood serum contained IgM to the rubella for instance fungi toxins and some antibiotics virus. What is this result indicative of? can inhibit activity of RNA-polymerase. A. Of exacerbation of a chronic disease What process will be disturbed in a cell in B. Of recurring infection with rubella virus case of inhibition of this enzyme? C. Of a chronic process A. Reparation D. The woman is healthy B. Translation E. Of primary infection C. Transcription 167. A boy found a spider with the following D. Replication morphological characteristics: it is 2 cm E. Processing long, has roundish black abdomen with two 162. As a result of destruction of certain rows of red spots on its dorsal side; four pairs brainstem structures an animal has lost of jointed limbs are covered with small its orientative reflexes in response to black hairs. What arthropod is it? strong photic stimuli. What structures were A. Mite destroyed? B. Solpuga A. Posterior tubercles of quadrigeminal plate C. Karakurt spider B. Anterior tubercles of quadrigeminal plate D. Scorpion C. Red nuclei E. Tarantula D. Black substance 168. Examination of a young woman E. Vestibular nuclei revealed a node-like, soft and elastic 163. Inflammation of the tympanic cavity homogenous tumour of pinkish-white (purulent otitis media) was complicated colour along the acoustic nerve. The tumour by inflammation of mammillary process contains cell bundles with oval nuclei. sockets. What wall of tympanic cavity did Cellular fibrous bundles form rhythmic the pus penetrate into the sockets through? structures made up by parallel rows of A. Lateral regularly oriented cells arranged in form B. Medial of a palisade with cell-free homogenous C. Anterior zone (Verocay bodies) between them. What D. Posterior tumour is it? E. Superior A. Neurinoma 164. A 35 year old man got an injury that B. Ganglioneuroblastoma
  • 18. C. Malignant neurinoma chromophilic and chromophobic. Identify D. Neuroblastoma this organ: E. Ganglioneurinoma A. Hypothalamus 169. Two hours after an exam a student had B. Adrenal glands a blood count done and it was revealed C. Hypophysis that he had leukocytosis without significant D. Thyroid gland leukogram modifications. What is the E. Parathyroid gland most probable mechanism of leukocytosis 174. A patient came to the hospital complaining development? about quick fatigability and apparent A. Redistribution of leukocytes in the organism muscle weakness. Examination revealed B. Deceleration of leukocyte migration to an autoimmune disease that causes disorder the tissues of functional receptor condition in C. Leukopoiesis intensification and neuromuscular synapses. What transmitter deceleration of leukocyte lysis will be blocked? D. Deceleration of leukocyte lysis A. Noradrenalin E. Leukopoiesis intensification B. Serotonin 170. If a man has an attack of bronchiospasm C. Glyeine it is necessary to reduce the effect of D. Dopamine vagus on smooth muscles of bronchi. What E. Acetylcholine membrane cytoreceptors should be blocked 175. Microscopic examination of a Gramstained for this purpose? scrape from patient's tongue A. M-cholinoreceptors revealed oval, round, elongated chains of B. β-adrenoreceptors dark-violet gemmating cells. What disease C. α- and β-adrenoreceptors can be caused by this causative agent? D. α-adrenoreceptors A. Staphylococcic infection E. .N-cholinoreceptors B. Streptococcic infection 171. Plasmic factors of blood coagulation C. Diphtheria are exposed to post-translational modification D. Actinomycosis with the participation of vitamin K. It E. Candidosis is necessary as a cofactor in the enzyme 176. A patient ill with chronic gastritis went system of γ-carboxylation of protein factors for endogastric pH-metry that allowed to cif blood coagulation due to the increased reveal decreased acidity of gastric juice. It affinity of their molecules with calcium ions. is indicative of diminished function of the What amino acid is carboxylated in these following cells: proteins? A. Parietal exocrinocytes A. Valine B. Accessory cells B. Arginine C. Endocrinocytes C. Phenylalanine D. Cervical cells D. Glutamic E. Chief exocrinocytes E. Serine 177. Examination of a 60 year old patient 172. A sensitive neural ganglion consists of revealed hyperglycemia and glucosuria. A roundish neurocytes with one extension that doctor administered him a medication for divides into axon and dendrite at some distance internal use. What medication is it? from the perikaryon. What are these A. Pancreatine cells called? B. Furosemide A. Multipolar C. Oxytocin B. Pseudounipolar D. Glibenclamid C. Unipolar E. Corglycon D. Apolar 178. A 6 month old baby ill with bronchitis E. Bipolar was taken for an X-ray of chest. Apart 173. An endocrinal gland with parenchyma of changes associated with bronchi the X-ray consisting of epithelium and neural tissue film showed a shadow of thymus gland. is under morphological examination. Epithelial What might have caused such changes? trabecules have two types of cells: A. It is caused by thymus inflammation
  • 19. B. It is caused by abnormal position rhinitis and loss of sense of smell for a week. C. It is caused by neoplastic process Objectively: nasal cavity contains a lot of D. The above-mentioned condition is a mucus that covers mucous membrane and normal variant for this age blocks olfactory receptors. In what part of E. It's the effect of bronchitis nasal cavity are these receptors situated? 179. A 20 year old patient complains of A. Common nasal meatus general weakness, dizziness, quick fatigability. B. Vestibule of nose Blood analysis results: Hb- 80 C Median nasal turbinate g/l. Microscopical examination results: D. Inferior nasal turbinate erythrocytes are of modified form. This E. Superior nasal turbinate condition might be caused by: 185. A cardiac electric stimulator was A. Obturative jaundice implanted to a 75 year old man with heart B. Acute intermittent porphyria rate of 40 bpm. Thereafter the heart rate C. Sickle-cell anemia rose up to 70 bpm. The electric stimulator D. Addison's disease has undertaken the function of the following E. Hepatocellular jaundice heart part: 180. In course of laparotomy a surgeon A. His' bundle branches revealed gangrenous lesion of descending B. Sinoatrial node colon. It was caused by thrombosis of the C. His' bundle fibel's following artery: D. Purkinje's fibers A. Dexter colic E. Atrioventricular node B. Median colic 186. Blood of a patient with presumable C. Ileocolic sepsis was inoculated into sugar broth. D. Sinister colic There appeared bottom sediment. Repeated E. Superior mesenteric artery inoculation into blood agar caused growth 181. In course of a small pelvis operation it of small transparent round colonies became necessary to ligate an ovarian artery. surrounded by hemolysis zone. Examination What formation may be accidentally ligated of a smear from the sediment revealed together with it? gram-positive cocci in form of long chains. A. Round ligament of uterus What microorganisms are present in blood B. Urethra ~ of this patient? C. Uterine tube A. Streptococci D. Internal iliac vein B. Tetracocci E. Ureter C. Sarcina 182. A patient has a deep cut wound on the D. Staphylococci posterior surface of his shoulder in its middle E. Micrococci third. What muscle might he injured? 187. A patient has low rate of magnesium A. Anconeus muscle ions that are necessary for affixion of ribosomes B. Biceps muscle of arm to the endoplasmic reticulum. It is C Brachial muscle known that it causes disturbance of protein D. Coracohrachial muscle biosynthesis. At what stage is protein E. Triceps muscle of arm biosynthesis 183. A patient was admitted to the surgical impaired? department with suspected inflammation of A. Termination Meckel`s diverticulum. What part of bowels B. Translation should be examined in order to discover the C. Amino acid activation diverticulum in course of an operation? D. Replication A. Ileum E. Transcription B. Jejunmum 188. A patient with hypertensic crisis was C Colon ascendens admitted to the cardiological department, D. Duodenum he was injected intravenously with an E. Caecum antihypertensive 184. A 35 year old patient applied to a drug - salt of an alkaline-earth doctor with complaints about having intense metal. What drug was injected?
  • 20. A. Benzohexamethonium C. Louis-Bar B. Potassium chloride D. Wiskott-Aldrich C Sodium hydrocarbonate E. DiGeorge D. Magnesium sulfate 195. A woman suffering from osteochondrosis E. Caleium lactate felt acute pain in her humeral articulation 189. Examination of a 35 year old patient that became stronger when she abducted her revealed high acidity of gastric juice. What shoulder. These symptoms might be caused receptors should be blocked in order to by damage of the following nerve: reduce it? A. Dorsal scapular nerve A. α 1-adrenoreceptors B. Axillary nerve B. β2-adrenoreceptors C. Subscapular nerve C. β1 -adrenoreceptors D. Subclavicular nerve D. α2-adrenoreceptors E. Throracodorsal nerve E. Histamine 196. A boy has I (I0 I0) blood group and 190. While performing an inguinal canal his sister has IV (IA IB) blood group. What operation on account of hernia a surgeon blood groups do their parents have? damaged the canal's contents. What exactly A. III (IB I0) and IV (IA IB) was damaged? B. II (IA IA) and III (IB I0) A. Funiculus spermaticus C. II (IA I0) and III (IB I0) B. Lig. teres uteri D. I (I0 I0) and III (IB I0) C. - E. I (I0 I0) and IV (IA IB) D. Lig. inguinale 197. A patient has a cluster of matted E. Urarchus together dense lymph nodes on his neck. 191. A 3 year old child with fever was given Histological aspirin. It resulted in intensified erythrocyte examination of a removed lymph haemolysis. Hemolytic anemia might have node revealed proliferation of reticular cells, been caused by congenital insufficiency of presense of Reed-Sternberg cells. What disease the following enzyme: is meant? A. Glycogen phosphorylase A. Myeloblastic leukosis B. Glucose 6-phosphate dehydrogenase B. Lyimphogranulomatosis C. Glycerol phosphate dehydrogenase C. Lymphoblastic leukosis D. Glucose 6-phosphatase D. Myelocytic leukosis E. γ-glutamiltransferase E. Lympholcytic leukosis 192. After consumption of rich food a patient 198. A woman underwent an operation on has nausea and heartburn, steatorrhea. account of extrauterine (tubal) pregnancy. This condition might he caused by: In course of the operation the surgeon A. Increased lipase secretion should ligate the branches of the following B. Bile acid deficiency arteries: C. Amylase deficiency A. Uterine and superior cystic D. Disturbed phospholipase synthesis B. Superior cystic and ovarian E. Disturbed tripsin synthesis C. Uterine and inferior cystic 193. A patient complains of dizziness and D. Uterine and ovarian hearing loss. What nerve is damaged? E. Inferior cystic and ovarian A. Vestibulocochlear 199. A specimen stained by Ozheshko B. Vagus method contains rod-like microorganisms C. Sublingual stained blue with round terminal D. Trochlear components stained red. What are these E. Trigeminus components called? 194. A child was born with cleft palate. A. Capsules Examination revealed aorta defects and B. Cilia reduced number of T-Iymphocytes in blood. C. Mesosomas What immunodeficient syndrome is it? D. Spores A. Swiss-type E. Flagella B. Chediak-Higashi 200. A 37 year old patient suffering from
  • 21. obliterating vascular endarteritis of lower limbs takes daily 60 microgram/kilogram of phenylin. Because of presentations of convulsive disorder (craniocerebral trauma in anamnesis) he was prescribed phenobarbital. Withholding this drug caused nasal hemorhage. What is this complication connected with? A. Aliphatic hydroxylation of phenobarbital B. Inhibition of microsomal oxidation in liver caused by phenobarbital C. Induction of enzymes of microsomal oxidation in liver caused by phenobarbital D. Conjugation of phenylin with glucuronic acid E. Oxidative deamination of phenylin